Pancreas Flashcards

1
Q

What is the duct of Wirsung?

A

Major pancreatic duct that forms in the pancreatic head and descends inferiorly and joins the intrapancreatic portion of the common bile duct to form the common pancreaticobiliary channel proximal to the ampulla of Vater

How well did you know this?
1
Not at all
2
3
4
5
Perfectly
2
Q

What is the duct of Santorini?

A

Accessory pancreatic duct that drains the anterior portion of the pancreatic head

How well did you know this?
1
Not at all
2
3
4
5
Perfectly
3
Q

What is the blood supply to the head of the pancreas?

A

Anterior and posterosuperior pancreaticoduodenal arteries from the gastroduodenal artery that form collaterals with branches of the superior mesenteric artery (SMA) (inferoanterior and posterior pancreaticoduodenal arteries)

How well did you know this?
1
Not at all
2
3
4
5
Perfectly
4
Q

What is the venous drainage?

A

It parallels the arterial supply; drains into the portal system via the superior mesenteric and splenic veins

How well did you know this?
1
Not at all
2
3
4
5
Perfectly
5
Q

Which enzyme is responsible for pancreatic necrosis in the presence of bile?

A

Phospholipase A

How well did you know this?
1
Not at all
2
3
4
5
Perfectly
6
Q

What defines a high-output pancreatic fistula?

A

Output in excess of 200 mL/day

How well did you know this?
1
Not at all
2
3
4
5
Perfectly
7
Q

What are the etiologies of acute pancreatitis?

A

Gallstones and alcohol account for >90% of cases. Other causes include hyperlipidemia, hypercalcemia, trauma, pancreatic duct obstruction, ischemia, drugs, familial, and idiopathic.

How well did you know this?
1
Not at all
2
3
4
5
Perfectly
8
Q

What are some common medications implicated as possible etiologies of pancreatitis?

A
Azathioprine
Furosemide
Thiazides
Sulfonamide
Tetracycline
Steroids
Estrogens
Ethacrynic acid, and 
H2 blockers
How well did you know this?
1
Not at all
2
3
4
5
Perfectly
9
Q

What metabolic conditions could cause pancreatitis?

A

Hyperlipidemia (types I, IV, and V have been implicated);
Hypercalcemia, which is most commonly found with hyperparathyroidism that could lead to intraductal precipitation of calcium

How well did you know this?
1
Not at all
2
3
4
5
Perfectly
10
Q

How is acute pancreatitis diagnosed?

A

The diagnosis of pancreatitis requires 2 of the following 3 features:

1) Abdominal pain characteristic of acute pancreatitis
2) A serum amylase or lipase level at least 3 times the upper limit of normal, and
3) Characteristic findings of acute pancreatitis on computed tomography (CT)

How well did you know this?
1
Not at all
2
3
4
5
Perfectly
11
Q

Which enzyme is implicated in the etiology of pancreatitis?

A

Trypsin

How well did you know this?
1
Not at all
2
3
4
5
Perfectly
12
Q

Which serum enzyme rises within 2 hours of the onset of pancreatitis and peaks within 48 hours?

A

Amylase

How well did you know this?
1
Not at all
2
3
4
5
Perfectly
13
Q

What antibiotics are indicated for patients with mild pancreatitis?

A

None! Antibiotics neither improve the course nor prevent septic complications.

How well did you know this?
1
Not at all
2
3
4
5
Perfectly
14
Q

What CT scan findings are suggestive of chronic pancreatitis?

A

Dilated pancreatic duct, calcifications, and parenchymal atrophy

How well did you know this?
1
Not at all
2
3
4
5
Perfectly
15
Q

What are the early Ranson criteria (on admission)?

A
Glucose > 200 mg/dL, 
age >55, 
LDH > 350 IU/L, 
AST > 250 IU /L, 
WBC > 16,000
How well did you know this?
1
Not at all
2
3
4
5
Perfectly
16
Q

What are the late Ranson criteria (48 hours)?

A
Calcium <8.0 mg/dL, 
HCT drop > 10%, 
Pa0 2 <60 mm Hg, 
BUN increase by 5 or more mg/dL, 
base deficit >4 mEq/L, 
fluid sequestration >6 L
How well did you know this?
1
Not at all
2
3
4
5
Perfectly
17
Q

How do Ranson criteria predict mortality?

A

0 to 2 signs, 2%
3 to 4 signs, 15%
5 to 6 signs, 40%
7 to 8 signs, -100%

How well did you know this?
1
Not at all
2
3
4
5
Perfectly
18
Q

What are the indications for surgery in chronic pancreatitis?

A
Intractable abdominal pain, 
common bile duct obstruction, 
duodenal obstruction, 
persistent pseudocysts, 
pancreatic fistula or ascites, 
variceal hemorrhage secondary to splenic vein obstruction (treated by splenectomy), to rule out pancreatic malignancy, 
colonic obstruction
How well did you know this?
1
Not at all
2
3
4
5
Perfectly
19
Q

What are possible complications of pancreatitis?

A

Pancreatic necrosis, pseudocyst, pancreatic fistulas, pseudoaneurysm, hemorrhage, pancreatic ascites, sepsis

How well did you know this?
1
Not at all
2
3
4
5
Perfectly
20
Q

How does chronic pancreatitis present?

A

Abdominal pain, diabetes, steatorrhea, and pancreatic calcification. Amylase is not typically elevated in chronic pancreatitis.

How well did you know this?
1
Not at all
2
3
4
5
Perfectly
21
Q

Initial management of pancreatic duct stricture from chronic pancreatitis:

A

Pancreatic duct stenting

How well did you know this?
1
Not at all
2
3
4
5
Perfectly
22
Q

What are some surgical procedures used in chronic pancreatitis?

A

Duval procedure (distal pancreatectomy with end-to-end pancreaticojejunostomy)

Puestow procedure (lateral side-to-side pancreaticojejunostomy), which is most widely used and preferred;

pancreatic resection, pancreatic denervation, islet cell transplantation (for type 1 diabetes mellitus)

Frey procedure (coring out of diseased portion of pancreatic head and then lateral pancreaticojejunostomy for chronic pancreatitis)

Beger procedure (duodenum-preserving pancreatic head resection)

How well did you know this?
1
Not at all
2
3
4
5
Perfectly
23
Q

When is a follow-up CT scan for pancreatitis indicated?

A

Clinical deterioration (pseudocyst, fluid collections, or necrosis)

How well did you know this?
1
Not at all
2
3
4
5
Perfectly
24
Q

Why does shock occur in severe pancreatitis?

A

Hypotension and subsequent shock are related to hemodynamic changes resembling sepsis rather than hypovolemia. Cardiac output is generally increased with decreased peripheral vascular resistance.

How well did you know this?
1
Not at all
2
3
4
5
Perfectly
25
Q

What are the 4 types of acute pancreatitis complications per the revised Atlanta Classification?

A
  1. Acute peripancreatic fluid collection or post-necrotic pancreatic/peripancreatic fluid collection. Both occur within 4 weeks and are subclassified as sterile or infected.
  2. Pancreatic pseudocyst or walled-off pancreatic necrosis. Both occur after 4 weeks and are subclassified as sterile or infected.

(Use of the terms acute pseudo cyst and pancreatic abscess is now discouraged.)

How well did you know this?
1
Not at all
2
3
4
5
Perfectly
26
Q

What percentage of pseudocysts spontaneously resolve within 4 to 6 weeks?

A

50%

How well did you know this?
1
Not at all
2
3
4
5
Perfectly
27
Q

How are pseudocysts managed?

A

Expectant, supportive management for 4 to 6 weeks If no resolution occurs, wait until a thick, fibrous wall has formed and perform internal cyst drainage via an open or endoscopic cystgastrostomy, cystojejunostomy, or cystduodenostomy

A biopsy should always be performed to rule out malignancy.

External drainage may be pursued for infected collections of pseudocysts.

How well did you know this?
1
Not at all
2
3
4
5
Perfectly
28
Q

How long does it take a pseudocyst to mature?

A

4 to 6 weeks

How well did you know this?
1
Not at all
2
3
4
5
Perfectly
29
Q

Indications for surgical intervention for pancreatic pseudocysts:

A

Pseudocyst has not resolved by 6 weeks and persistently greater than 6 cm

How well did you know this?
1
Not at all
2
3
4
5
Perfectly
30
Q

List the enteric methods of pseudocyst drainage:

A

Cystogastrostomy, cystoduodenostomy, Roux-en-Y cystojejunostomy, lateral pancreaticojejunostomy

How well did you know this?
1
Not at all
2
3
4
5
Perfectly
31
Q

What CT scan criteria are used for the diagnosis of pancreatic necrosis?

A

Well-demarcated areas of nonenhancing pancreatic tissue > 3 cm or occupying more than 30% of the gland

How well did you know this?
1
Not at all
2
3
4
5
Perfectly
32
Q

How is infected peripancreatic fluid collection or infected walled-off pancreatic necrosis diagnosed?

A

CT-guided percutaneous fine needle aspiration

How well did you know this?
1
Not at all
2
3
4
5
Perfectly
33
Q

After a prolonged course of drainage, what surgical options are there for debridement of pancreatic necrosis?

A

Open or video-assisted retroperitoneal debridement (VARD)

How well did you know this?
1
Not at all
2
3
4
5
Perfectly
34
Q

What antibiotics are indicated in pancreatic necrosis involving >30% of the gland?

A

Imipenem or meropenem

How well did you know this?
1
Not at all
2
3
4
5
Perfectly
35
Q

Aspiration of the necrotic pancreas is negative, now what?

A

Continue nonoperative management

How well did you know this?
1
Not at all
2
3
4
5
Perfectly
36
Q

How are pancreatic fistulas managed?

A

NPO, IV hydration, TPN, electrolyte replacement, and skin care.

Somatostatin has not been shown to accelerate closure rate. If no resolution, an endoscopic retrograde cholangiopancreatography (ERCP) to evaluate anatomy and ultimate surgical internal drainage or distal resection.

How well did you know this?
1
Not at all
2
3
4
5
Perfectly
37
Q

How does hemorrhage manifest in the setting of pancreatitis?

A

It usually is due to erosion of an arterial pseudoaneurysm secondary to pseudocyst or necrotizing pancreatitis.

Diagnosis is by angiography.

Immediate surgery is indicated should the patient become unstable.

Selective embolization may be possible in stable patients.

How well did you know this?
1
Not at all
2
3
4
5
Perfectly
38
Q

Which congenital anomaly results from failure of fusion of the dorsal and ventral pancreatic ducts?

A

Pancreas divisum

How well did you know this?
1
Not at all
2
3
4
5
Perfectly
39
Q

What is the most common benign neoplasm in the pancreas?

A

Serous cystadenoma. These tumors have a low rate of malignant transformation.

How well did you know this?
1
Not at all
2
3
4
5
Perfectly
40
Q

What is the treatment for serous cystadenoma?

A

Resection is generally recommended, but this lesion can be closely followed in high-operative-risk patients

How well did you know this?
1
Not at all
2
3
4
5
Perfectly
41
Q

What tumor exhibits sunburst central calcification on CT scan?

A

Serous cystadenoma

How well did you know this?
1
Not at all
2
3
4
5
Perfectly
42
Q

Overall 5-year survival rate for pancreatic cancer:

A

< 5% in general. Recent reports have cited a 5-year survival of 20% to 25%.

How well did you know this?
1
Not at all
2
3
4
5
Perfectly
43
Q

Median survival time of pancreatic cancer patients:

A

4to 6months

How well did you know this?
1
Not at all
2
3
4
5
Perfectly
44
Q

What is the most significant modifiable risk factor for pancreatic cancer?

A

Cigarette smoking

How well did you know this?
1
Not at all
2
3
4
5
Perfectly
45
Q

What are the presenting signs of pancreatic cancer?

A

Abdominal pain, jaundice (could be painless jaundice, but this is actually more commonly associated with ampullary and duodenal neoplasms), and weight loss

How well did you know this?
1
Not at all
2
3
4
5
Perfectly
46
Q

What imaging studies are needed for diagnosis?

A

Ultrasound to evaluate biliary anatomy, contracted CT scan

How well did you know this?
1
Not at all
2
3
4
5
Perfectly
47
Q

What are some inherited disorders that increase the risk of pancreatic cancer?

A
MEN
Hereditary pancreatitis
Familial Adenomatous Polyposis
Hereditary Non-Polyposis Colorectal Cancer
Von Hippel-Lindau and Gardner syndromes
How well did you know this?
1
Not at all
2
3
4
5
Perfectly
48
Q

What serologic tumor markers are measured?

A

CA 19-9 (sensitivity 83%, specificity 82%),

CEA (sensitivity 56%, specificity 75%)

How well did you know this?
1
Not at all
2
3
4
5
Perfectly
49
Q

What imaging modality is beneficial in assessing the T stage of the tumor in pancreatic cancer?

A

Endoscopic ultrasonography

How well did you know this?
1
Not at all
2
3
4
5
Perfectly
50
Q

Which chemotherapeutic agents are commonly used in adjuvant therapy?

A

The 2 most active agents are 5-fluorouracil and gemcitabine. Mitomycin C, streptozocin, doxorubicin, and lomustine have also been used. 5- FU potentiates radiation therapy.

How well did you know this?
1
Not at all
2
3
4
5
Perfectly
51
Q

FDA approved for combination with gemcitabine for first-line treatment of locally advanced, unresectable, or metastatic pancreatic cancer:

A

Erlotinib (Tarceva)

How well did you know this?
1
Not at all
2
3
4
5
Perfectly
52
Q

What percentage of patients with pancreatic cancer will have had a new diagnosis of diabetes?

A

20%

How well did you know this?
1
Not at all
2
3
4
5
Perfectly
53
Q

Tl pancreatic cancer:

A

Tumor limited to the pancreas, <2 cm

How well did you know this?
1
Not at all
2
3
4
5
Perfectly
54
Q

T2 pancreatic cancer:

A

Tumor limited to the pancreas, >2 cm

How well did you know this?
1
Not at all
2
3
4
5
Perfectly
55
Q

T3 pancreatic cancer:

A

Tumor extends beyond the pancreas but without involvement of the celiac axis or the SMA

How well did you know this?
1
Not at all
2
3
4
5
Perfectly
56
Q

T4 pancreatic cancer:

A

Tumor involves the celiac axis or the SMA

How well did you know this?
1
Not at all
2
3
4
5
Perfectly
57
Q

What T stage denotes unresectable disease?

A

T4, by definition

How well did you know this?
1
Not at all
2
3
4
5
Perfectly
58
Q

What reconstruction is performed during standard pancreaticoduodenectomy (Whipple procedure)?

A

End-to-side pancreaticojejunostomy, hepaticojejunostomy, gastrojejunostomy

How well did you know this?
1
Not at all
2
3
4
5
Perfectly
59
Q

What percentage of pancreatic neoplasms are mucinous cystic neoplasms?

A

2%

How well did you know this?
1
Not at all
2
3
4
5
Perfectly
60
Q

What distinguishes mucinous cystic neoplasm from intraductal papillary mucinous neoplasm?

A

Mucinous cystic neoplasm rarely communicates with the main pancreatic duct

How well did you know this?
1
Not at all
2
3
4
5
Perfectly
61
Q

Should a mucinous cystic neoplasm be resected?

A

Yes. because of malignant potential

How well did you know this?
1
Not at all
2
3
4
5
Perfectly
62
Q

What rash is seen with glucagonoma?

A

Necrolytic migratory erythema

How well did you know this?
1
Not at all
2
3
4
5
Perfectly
63
Q

From which islet cells do glucagonoma arise?

A

Alpha cells

Glucagonomas are usually malignant and metastatic at diagnosis.
Tx: Debulking surgery

How well did you know this?
1
Not at all
2
3
4
5
Perfectly
64
Q

What laboratory test confirms the diagnosis of glucagonoma?

A

Elevated plasma glucagon >150 pg/mL (many will have levels in excess of 500 pg/mL)

How well did you know this?
1
Not at all
2
3
4
5
Perfectly
65
Q

Which test is used for localization of glucagonoma?

A

CT or MRI

How well did you know this?
1
Not at all
2
3
4
5
Perfectly
66
Q

What is another name for VIPoma?

A

Verner Morrison syndrome

How well did you know this?
1
Not at all
2
3
4
5
Perfectly
67
Q

What triad is associated with VIPoma?

A

Watery diarrhea, hypokalemia, and achlorhydria

How well did you know this?
1
Not at all
2
3
4
5
Perfectly
68
Q

What tests are used for localization of VIPoma?

A

Ultrasound, CT (often tumor is 3 cm or larger), angiography, and transhepatic venous sampling may be used as second-line in difficult cases
Often found at the tail

How well did you know this?
1
Not at all
2
3
4
5
Perfectly
69
Q

What is the appropriate treatment ofVIPoma?

A

Enucleation or surgical resection, depending on location

Debulking as palliative treatment

How well did you know this?
1
Not at all
2
3
4
5
Perfectly
70
Q

Where are 85% of all gastrinomas located?

A

In the gastrinoma triangle, the borders are the pancreatic neck/body junction, confluence of cystic and common hepatic ducts, and the junction of second and third portions of the duodenum

How well did you know this?
1
Not at all
2
3
4
5
Perfectly
71
Q

What percentage of islet cell tumors do insulinomas comprise?

A

25%

How well did you know this?
1
Not at all
2
3
4
5
Perfectly
72
Q

How can this condition be distinguished from factitious hyperinsulinemia?

A

C-peptide:insulin ratio is 1:1 in insulinoma

How well did you know this?
1
Not at all
2
3
4
5
Perfectly
73
Q

What percentage of insulinomas are malignant?

A

10%

How well did you know this?
1
Not at all
2
3
4
5
Perfectly
74
Q

What triad is suggestive of the diagnosis of insulinoma?

A

Whipple Triad:

1) symptoms of hypoglycemia with fasting
2) blood glucose <50 mg/dL
3) relief with glucose intake

How well did you know this?
1
Not at all
2
3
4
5
Perfectly
75
Q

What test is diagnostic for insulinoma?

A

72-hour fast. Insulin and glucose are measured every 6 hours. Symptoms of hypoglycemia develop in 12 hours.

Insulin:glucose ratio >0.3 or serum insulin >6 µU/mL is diagnostic.

How well did you know this?
1
Not at all
2
3
4
5
Perfectly
76
Q

What is the surgical procedure of choice for insulinoma?

A

Enucleation

Exceptions:

1) Close to the main pancreatic duct
2) >2cm
- -> Whipple procedure

How well did you know this?
1
Not at all
2
3
4
5
Perfectly
77
Q

What are the 4Ds of glucagonoma?

A

Diabetes
Dermatitis
DVT
Depression

How well did you know this?
1
Not at all
2
3
4
5
Perfectly
78
Q

What percentage of patients with glucagonoma have diabetes?

A

> 75%

How well did you know this?
1
Not at all
2
3
4
5
Perfectly
79
Q

What does VIP in VIPoma stand for?

A

Vasoactive intestinal peptide

How well did you know this?
1
Not at all
2
3
4
5
Perfectly
80
Q

The diagnosis of VIPoma is supported by what serum VIP level?

A

> 200pg/mL

How well did you know this?
1
Not at all
2
3
4
5
Perfectly
81
Q

Management of the diarrhea of VIPoma:

A

Octreotide

How well did you know this?
1
Not at all
2
3
4
5
Perfectly
82
Q

What is the rarest pancreatic islet cell tumor?

A

Somatostatinoma

How well did you know this?
1
Not at all
2
3
4
5
Perfectly
83
Q

What somatostatin level confirms the diagnosis of somatostatinoma?

A

> 10ng/mL

How well did you know this?
1
Not at all
2
3
4
5
Perfectly
84
Q

What study should be used to define the extent of pancreatic islet cell tumors?

A

Octreotide scan

How well did you know this?
1
Not at all
2
3
4
5
Perfectly
85
Q

What is the most malignant pancreatic endocrine tumor?

A

Gastrinoma

How well did you know this?
1
Not at all
2
3
4
5
Perfectly
86
Q

What is the most benign pancreatic endocrine tumor?

A

Insulinoma

How well did you know this?
1
Not at all
2
3
4
5
Perfectly
87
Q

What is the surgical management of endocrine tumors in the tail of the pancreas?

A

Distal pancreatectomy

How well did you know this?
1
Not at all
2
3
4
5
Perfectly
88
Q

What is the surgical management of endocrine tumors in the head of the pancreas?

A

Whipple procedure

How well did you know this?
1
Not at all
2
3
4
5
Perfectly
89
Q

This receptor is found on many pancreatic islet cell tumors:

A

Somatostatin receptor

How well did you know this?
1
Not at all
2
3
4
5
Perfectly
90
Q

Standard supportive measures for patients with mild pancreatitis include the ff:

A. IV fluid and electrolyte therapy

B. Withholding of analgesics to allow serial abdominal examinations

C. Nasogastric decompression

D. Massive antibiotics

E. Percutaneous drainage

A

A. IV fluid and electrolyte therapy

How well did you know this?
1
Not at all
2
3
4
5
Perfectly
91
Q

A 52-year-old male, known alcoholic, is evaluated because of chronic abdominal pain. The clinical diagnosis of chronic pancreatitis is supported by ERCP findings of pancreatic ductal ectasia with alternating areas of stricture and dilatation. Several pancreatic ductal stones are also noted. With chronic pain as the operative indication, the most appropriate procedure would be:

A. 90% distal pancreatectomy

B. Longitudinal pancreaticojejunostomy

C. Distal pancreatectomy with end pancreatic jejunostomy

D. Total pancreatectomy

E. ERCP and extraction of stones via ampulla of Vater

A

B. Longitudinal pancreaticojejunostomy

How well did you know this?
1
Not at all
2
3
4
5
Perfectly
92
Q

Which of the following is the most common cause of obstructive jaundice in patients with chronic pancreatitis?

A. Adenocarcinoma of the head of the pancreas

B. Choledocholithiasis

C. Fibrotic stricture of the CBD

D. Pseudocyst formation

E. Mirrizi’s syndrome

A

C. Fibrotic stricture of the CBD

How well did you know this?
1
Not at all
2
3
4
5
Perfectly
93
Q

A 72-year-old man develops jaundice and is demonstrated to have a 2.5cm mass in the pancreatic head by CT. There are no signs of unresectability on CT examination. Fine needle aspiration cytology is positive for adenocarcinoma. Which of the following intraoperative findings would indicated unresectability?

A. Fibrotic reaction of the body and tail of the pancreas

B. Microscopic tumor cells in the greater omentum on frozen section

C. Tumor abutting the superior mesenteric vein

D. Cholelithiasis

E. History of pancreatitis 6 months ago

A

B. Microscopic tumor cells in the greater omentum on frozen section

How well did you know this?
1
Not at all
2
3
4
5
Perfectly
94
Q

Which of the following surgical procedures has the lowest incidence of recurrent jaundice when used in the context of unresectable carcinoma of the head of the pancreas?

A. Choledochoduodenostomy

B. Cholecystojejunostomy

C. Cholecystoduodenostomy

D. Choledochojejunostomy

E. None of the above

A

D. Choledochojejunostomy

How well did you know this?
1
Not at all
2
3
4
5
Perfectly
95
Q

A 45-year-old woman is evaluated for epigastric and back pain. PE is normal. CT of the abdomen reveals an 8cm cystic lesion in the region of the tail of the pancreas. The cyst demonstrates internal separations and papillary projections from its walls. Which of the following diagnoses is most likely in this patient?

A. Pancreatic lymphoma

B. Retroperitoneal liposarcoma

C. Pancreatic pseudocyst

D. Pancreatic mucinous cystadenoma

E. Benign cyst

A

D. Pancreatic mucinous cystadenoma

How well did you know this?
1
Not at all
2
3
4
5
Perfectly
96
Q

Treatment for the above patient is:

A. Chemotherapy with or without radiation

B. Core needle aspiration biopsy

C. Drainage procedures

D. Resection

E. Aspiration

A

D. Resection

How well did you know this?
1
Not at all
2
3
4
5
Perfectly
97
Q

The patient with jaundice is suspected of having complete distal CBD tumor. The best procedure for determining the extent of obstruction is?

A. Endoscopic ultrasonography

B. ERCP

C. Percutaneous transhepatic biliary drainage

D. HIDA scan

E. MRCP

A

E. MRCP

How well did you know this?
1
Not at all
2
3
4
5
Perfectly
98
Q

The patient has an unresectable Klatskin tumor seen in CT scan. The patient complains of jaundice. What is the best procedure to do?

A. ERCP

B. PET CT Scan

C. Percutaneous Transhepatic Biliary Drainage

D. ERCP with sphincterotomy

E. Protime determination

A

E. Protime determination

How well did you know this?
1
Not at all
2
3
4
5
Perfectly
99
Q

A 56-year-old male has a 5cm periampullary tumor diagnosed by CT scan. There are no associated signs and symptoms except for a shallow jaundice. The next step is:

A. PET CT scan

B. ERCP and biopsy

C. Endoscopic ultrasonography

D. Chest xray

E. Alpha fetoprotein determination

A

B. ERCP and biopsy

How well did you know this?
1
Not at all
2
3
4
5
Perfectly
100
Q

A patient developed a pancreatic fistula after a pancreaticoduodenectomy. This was determined through the JP output. Drainage is 200ml/day. Appropriate treatment is

A. TPN

B. Breakdown of anastomosis and creation of a new one

C. PTBD

D. Insertion of drains in the 4 quadrants and lavage

E. Patient controlled analgesia

A

A. TPN

How well did you know this?
1
Not at all
2
3
4
5
Perfectly
101
Q

A 65-year-old female will undergo a Whipple procedure for an 8cm pancreatic head adenocarcinoma. CT scan of whole abdomen showed no other tumors. Appropriate procedure before doing the surgery is:

A. Transfuse blood in anticipation of blood loss during surgery.

B. Preoperative jejunostomy tube insertion for feeding.

C. Intraoperative liver ultrasound

D. Preoperative radiation therapy

E. Transarterial chemoembolization

A

C. Intraoperative liver ultrasound

102
Q

A 37-year-old woman has a 5cm choledochal cyst of the proximal common bile duct by ultrasonography. You should:

A. Request for a PET CT scan study.

B. Observe patient and repeat ultrasonography after 6 months.

C. ERCP and sphincterotomy

D. Cholecystectomy with IOC.

E. Resection, cholecystectomy, and hepaticojejunostomy

A

E. Resection, cholecystectomy, and hepaticojejunostomy

103
Q

A 45-year-old male developed an 8cm symptomatic pancreatic pseudocyst inferior to the stomach and located in the pancreatic head. You should:

A. Resect the cyst

B. Observe and do ultrasonography after 6 weeks

C. Drain the cyst with a JP drain

D. Internal drainage procedure

E. Whipple procedure

A

D. Internal drainage procedure

104
Q

A 91-year-old was diagnosed to have a painful pancreatic head adenocarcinoma after ERCP and biopsy. CT scan of the abdomen revealed an 11cm tumor. There are also multiple metastatic nodules in the liver. Next step is:

A. Serial CA 19-9 determination

B. PET CT scan

C. Chemotherapy and IMRT treatment of the primary tumor

D. Radiofrequency ablation of the pancreatic and liver tumors

E. Analgesia

A

E. Analgesia

105
Q

A 37-year-old male has on and off RUQ pain. Ultrasonography revealed multiple stones and thickened gallbladder wall. He does not want to undergo surgery unless there is a more concrete laboratory examination of cholecystitis. You would:

A. Request for SGPT

B. Alkaline phosphatase level

C. LDH level

D. HIDA scan

E. CEA determination

A

D. HIDA scan

106
Q

A 44-year-old male has an 8cm mass in the RUQ believed to be gallbladder in origin. It appears malignant by CT scan and invades to the inferior portion of the liver and right hepatic artery. There is also massive ascites. There is jaundice and pain. The next step is:

A. Insertion of a pigtail catheter and cell cytology of drainage

B. Chemotherapy

C. Radiation therapy

D. CA 19-9 determination

E. Combination chemotherapy and radiation therapy

A

A. Insertion of a pigtail catheter and cell cytology of drainage

107
Q

Which of the following can cause pigment stones in the bile duct?

A. Clonorchis sinensis

B. Ascaris lumbricoides

C. Stricture

D. Sclerosing cholangitis

E. All of the above

A

E. All of the above

108
Q

A 65-year-old woman developed high fever, jaundice, and RUQ pain. She was diagnosed with gallstones previously. What will be your next step?

A. MRCP

B. ERCP and possible extraction of stones

C. Cholecystectomy and IOC

D. Antibiotic treatment

E. Cholecystectomy and choledochoduodenostomy

A

D. Antibiotic treatment

109
Q

Neoplasms of the endocrine pancreas are relatively uncommon but do occur with enough frequency. Unfortunately, most pancreatic endocrine tumors are malignant, but the course of the disease is far more favorable than exocrine cancer. Which is true regarding pancreatic endocrine tumors?

A. Insulinomas are the most common pancreatic endocrine neoplasm.

B. Majority of insulinomas are malignant (80%)

C. Glucagonomas are more often in the head and neck of the pancreas and tend to be large tumors.

D. VIPoma syndrome is also known as Whipple’s triad consisting of fasting hypoglycemia, serum glucose level <50mg/l and relief of symptoms after administration of glucose.

A

A. Insulinomas are the most common pancreatic endocrine neoplasm.

110
Q

The blood supply to the pancreas comes from multiple branches from the celiac and superior mesenteric arteries. Which of the following is true regarding pancreatic vascular anatomy?

A. The gastroduodenal artery (GDA) becomes the superior pancreatidodeuodenal artery as it passes behind the first portion of the duodenum

B. As the superior mesenteric artery passes behind the neck of the pancreas, it gives off the inferior mesenteric artery at the inferior margin of the pancreas.

C. The venous drainage of the pancreas follows a pattern similar to the arterial supply, the veins are usually deeper to the arteries within the parenchyma of the pancreas

D. The common hepatic artery gives rise to the GDA and is an end artery.

A

A. The gastroduodenal artery (GDA) becomes the superior pancreatidodeuodenal artery as it passes behind the first portion of the duodenum

111
Q

Which patient with pancreatic cancer would most likely be deemed unresectable?

A. A 43-year-old male with central pancreatic mass, CA 19-9 of 150 and tumor abutting the splenic vein.

B. A 54-year-old female with a 3cm mass in the head of the pancreas, CA 19-9 of 150 and encasement of the superior mesenteric vein

C. 58-year-old female with central pancreatic mass, CA 19-9 of 200 and adhesion to the ventral surface of the IVC.

D. 67-year-old male with 5cm mass in the head of the pancreas, CA 19-9 OF 50 and encasement of the SMA

A

D. 67-year-old male with 5cm mass in the head of the pancreas, CA 19-9 OF 50 and encasement of the SMA

112
Q

A 67-year-old male was diagnosed with a resectable pancreatic head mass probably malignant. He underwent a pylorus-preserving pancreatiocoduodenectomy. Which is false?

A. Delayed gastric emptying is common after PPPD and is usually treated surgically.

B. The operative mortality rate for pancreaticduodenectomy has decreased to 5% in high volume centers.

C. The most common cause of death is sepsis, hemorrhage and cardiovascular events.

D. In DGE, IV Erythromycin may help but the problem usually resolves with time.

A

A. Delayed gastric emptying is common after PPPD and is usually treated surgically.

113
Q

The following statements are true of insulinomas:

A. 90% of insulinomas are sporadic and 10% are associated with MEN II syndrome.

B. Typical presentation is known as Whipple’s triad.

C. Serum insulin is elevated while C-peptide is decreased.

D. It is usually located in the triangle of Passaro.

A

B. Typical presentation is known as Whipple’s triad.

114
Q

A 42-year-old man who consumed more than 3 bottles of vodka weekly over the past 20 years is admitted with abdominal pain and hyperamylasemia due to pancreatitis. In determining his prognosis, which of the following factors would cause the greatest concern?

A. Hypercalcemia >12mg/dL

B. Age over 40 years

C. Hypoxemia

D. Hyperamylasemia >600U

A

C. Hypoxemia

115
Q

An 80-year-old male consults for a cystic tumor of the pancreatic head. ERCP was done showing mucin extruding from the ampulla of later. Diagnosis for this patient is:

A. Mucinous cystadenoma

B. Mucinous cystadenocarcinoma

C. Intraductal papillary neoplasm

D. Solid pseudopapillary tumor

A

C. Intraductal papillary neoplasm

116
Q

A 65-year-old male presents with a 2cm ampullary adenoma seen on endoscopy. Biopsy showed a benign adenoma. What is the best treatment option for the patient?

A. Duodenotomy and tumor excision with 2-3mm margins.

B. Pancreaticoduodenectomy

C. Observe

D. Repeat endoscopy after 2 months

A

A. Duodenotomy and tumor excision with 2-3mm margins

117
Q

Radiologic imaging has become the principal method of diagnosis of chronic pancreatitis. Which of the following is true?

A. ERCP has been considered as the most sensitive radiologic test for the diagnosis.

B. CT scan is the most sensitive and appropriate imaging once diagnosed

C. CT scan is highly reliable in ruling out pancreatic carcinoma when EUS findings are normal or equivocal, inconclusive

D. All patients should immediately undergo ERCP once diagnosed with pancreatitis.

A

A. ERCP has been considered as the most sensitive radiologic test for the diagnosis.

118
Q

The pancreas is innervated by the sympathetic and parasympathetic nervous system. Which among the following is true?

A. The pancreas has a poor supply of afferent sensory fibers, which is the main reason why painless jaundice occurs in malignancy.

B. The somatic fibers travel superiorly to the celiac ganglion and interruption of these fibers can stop pain transmission.

C. The parasympathetic system inhibits exocrine and endocrine function and the sympathetic system stimulates secretion.

D. The pancreas is located in the retroperitoneum which is responsible for the intense pain associated in advanced pancreatic cancer.

A

B. The somatic fibers travel superiorly to the celiac ganglion and interruption of these fibers can stop pain transmission.

119
Q

Ranson’s criteria (prognostic signs of pancreatitis) for acute pancreatitis not due to gallstones?

A
ADMISSION
- Age >55yo
- WBC >16,000
- Blood glucose >200mg/dL
Serum LDH >350 IU/L
Serum AST >250 U/dL

INITIAL 48H

  • Hematrocrit fall >10 points
  • BUN elevation >5mg/dL
  • Serum calcium <8 mg/dL
  • Arterial PO2 <60mmHg
  • Base deficit >4 mEq/L
  • Estimated fluid sequestration >6L

<3 positive criteria: mild, uncomplicated disease
>6 positive criteria: severe disease, 50% mortality

120
Q

Ranson’s criteria (prognostic signs of pancreatitis) for acute gallstone pancreatitis?

A
ADMISSION
Age >70yo
WBC >18,000
Blood glucose >220mg/dL
Serum LDH >400 IU/L
Serum AST >250 U/dL
During INITIAL 48H
Hematocrit fall >10 points
BUN elevation >2mg/dL
Serum calcium <8mg/dL
Base deficit >5mEq/L
Estimated fluid sequestration >4L

<3 positive criteria: mild, uncomplicated disease
>6 positive criteria: severe disease, 50% mortality

121
Q

Other approaches to predicting severity of acute pancreatitis?

A

24h of admission

  • APACHE II score of 8
  • CRP >150mg/dL
- BISAP
BUN >25mg/dL
Impaired mental status (GCS <15)
SIRS +
AGE >60
Pleural effusion +
122
Q

A gallstone etiology for pancreatitis is more likely in which subset of patients?

A
Females
>50yo
ALP>300
ALT>100
Amylase >4000
123
Q

Possible consequence of aggressive early enteral feeding In acute pancreatitis (before adequate resuscitation)?

A

Nonocclusive mesenteric ischemia

124
Q

Management of an acute noninfected pseudocyst?

A

Conservative

Half will resolve spontaneously

125
Q

Management of an acute symptomatic noninfected pseudocyst?

A

Pseudocysts persist because of communication with the main pancreatic duct and/or distal ductal stenosis.

EUS-guided internal drainage into stomach or duodenum or transpapillary stenting is the preferred approach.

Percutaneous drainage should be avoided because of the risk of external pancreatic fistula.

126
Q

Diagnosis of acute pancreatitis?

A
  • History of abdominal pain consistent with acute pancreatitis
  • > 3x elevation of pancreatic enzymes
  • CT scan if required to confirm diagnosis
127
Q

Initial assessment and management of acute pancreatitis?

A
  • Analgesia
  • Fluid resuscitation
  • Predict severity of pancreatitis
    • Ranson’s criteria
    • HAPS score
  • Assess systemic response
    • SIRS score
    • SOFA (organ failure)
128
Q

Reassessment and management of acute pancreatitis (4-6h)?

A

• Assess response to fluid resuscitation

1) mean arterial pressure
2) heart rate
3) urine output
4) hematocrit

• Determine etiology

1) Ultrasound for gallstones/sludge
2) History of alcohol consumption
3) Laboratory evaluation of other causes

• MRCP and/or Urgent ERCP if concomitant cholangitis is present

1) not for cholestasis or predicted severe disease per se

• Transfer to ICU or specialist center as needed

1) Deterioration or failure to respond to initial management
2) Intensive support for persistent organ failure

• Commence enteral nutrition

1) Once normovolemia restored (usually after 6 hours)
2) Commence via NG tube if no gastric stasis

• No prophylactic antibiotics or probiotics

129
Q

Conservative management and monitoring of acute pancreatitis?

A

• Clinical evaluation

1) Assess cardiovascular, respiratory, and renal function
2) Detect peritonitis and abdominal compartment syndrome

  • Daily C-reactive protein
  • Classify severity (mild, moderate, severe, critical)
  • Detect intolerance of NG EN

1) Advance tube for NJ feeding if needed
2) Consider supplemental parenteral nutrition by day 4

130
Q

Indications for pancreatic protocol CT scan?

A
  • For significant clinical deterioration and elevated CRP
  • For suspicion of local pancreatic complications
  • For suspected bowel ischemia
  • For acute bleeding (CTa) (if stable enough and consider embolization)
  • For abdominal compartment syndrome
131
Q

Invasive interventions in acute pancreatitis?

A
  • For deteriorating patient with suspected infected local complication
  • “Step up approach” with initial drain guided by current CT scan (percutaneous or endoscopic drainage)
  • Delay for 3 to 4 weeks with intensive care support, if possible
  • If failure to respond or secondary deterioration, repeat CT scan, and select appropriate minimally invasive technique based on available expertise and equipment

1) Video-assisted retroperitoneal debridement or percutaneous nephroscopic debridement
2) Endoscopic transluminal debridement
3) Ongoing large bore drainage and irrigation

132
Q

Indications for laparotomy?

A
  • Failed “step-up approach” for further debridement/ drainage
  • Acute abdomen (perforation or ischemia)
  • Severe abdominal compartment syndrome (rarely)
133
Q

Chronic inflammatory pancreatitis is characterized by?

A

Diffuse fibrosis
Loss of acinar elements
Predominant mononuclear cell infiltration

134
Q

Why does pancreas divisum predispose the pancreas to recurrent acute pancreatitis?

A

Pancreas divisum is the most common congenital anomaly involving the pancreas, and occurs in up to 10% of children.

It is thought to predispose the pancreas to recurrent acute pancreatitis and chronic pancreatitis, due to functional obstruction of a diminutive duct of Santorini that fails to communicate with Wirsung’s duct.

Endoscopic stunting through the lesser papilla may result in temporary relief of symptoms, and this response would increase the possibility that a permanent surgical or endoscopic intervention will be successful.

135
Q

What is autoimmune pancreatitis (AIP)?

A

A variant of chronic pancreatitis which is a non-obstructive, diffusely infiltrative disease associated with:

1) fibrosis
2) a mononuclear cell (lymphocyte, plasma cell, or eosinophil) infiltrate, and
3) an increased titer of one or more autoantibodies.

It is associated with illnesses with suspected or proven autoimmune etiology, such as Sjögren’s syndrome, rheumatoid arthritis, and type I DM.

AIP has two types:
I: Accompanying systemic or multiorgan dysfunction

II: Restricted to the pancreas

Compressive stenosis of the intrapancreatic portion of the common bile duct is frequently seen in both types of AIP, along with symptoms of obstructive jaundice.

Increased levels of serum β-globulin or immunoglobulin G4 are also present.

Steroid therapy is uniformly successful in ameliorating the disease, including any associated bile duct compression.

136 CFTR mutations that result in dislocation of the transmembrane protein have been found in AIP, and steroid therapy results in a normalization of the CFTR localization and a resumption of normal chloride and bicarbonate secretion.

136
Q

Histology features of early chronic pancreatitis?

A

Induration
Nodular scarring
Lobular regions of fibrosis

As the disease progresses, there is a loss of normal lobulation, with thicker sheets of fibrosis surrounding a reduced acinar cell mass and dilatation of ductular structures.

In severe chronic pancreatitis, there is considerable replacement of acinar tissue by broad, coalescing areas of fibrosis, and the islet size and number are reduced. Small arteries appear thickened, and neural trunks become prominent.

137
Q

What are pancreatic stones composed of?

A

Calcium carbonate crystals trapped in a matrix of fibrillar and other material. The fibrillar center of most stones contains no calcium but rather a mixture of other metals.

This suggests that stones form from an initial non calcified protein precipitate, which serves as a focus for layered calcium carbonate precipitation.

138
Q

Local complications of acute pancreatitis, <4 weeks (acute, no defined wall)?

A

Fluid: Acute pancreatic fluid collection (APFC)
Infected APFC

Solid with fluid: Acute necrotic collection (ANC)
Infected ANC

139
Q

Local complications of acute pancreatitis, >4 weeks (chronic, with a defined wall)?

A

Fluid: Pseudocyst
Infected pseudocyst

Solid with fluid: Walled off necrosis (WON)

140
Q

Role of radiologic imaging in chronic pancreatitis?

A

(a) diagnosis,
(b) the evaluation of severity of disease,
(c) detection of complications, and
(d) assistance in determining treatment options.

Transabdominal ultrasonography is frequently used as a screening method for patients with abdominal symptoms or trauma, and the extension of ultrasonic imaging to include endoscopic ultrasound (EUS) and laparoscopic US have resulted in the highest-resolution images that are capable of detecting very small (<1 cm) abnormalities in the pancreas.

EUS is now frequently used as a preliminary step in the evaluation of patients with pancreatic disease, and magnetic resonance cholangiopancreatography (MRCP) is increasingly being used to select patients who are candidates for the most invasive imaging method, ERCP

141
Q

Role of ultrasonography in chronic pancreatitis?

A

Ultrasonography is frequently used as an initial imaging method in patients with abdominal symptoms, and changes consistent with pancreatic duct dilatation, intraductal filling defects, cystic changes, and a heterogeneous texture are seen in chronic pancreatitis (Fig. 33-26).

The sensitivity of transabdominal ultrasonography ranges from 48% to 96%, and it is operator dependent.

However, the contour, texture, and ductal pattern are usually quite discernible, and it is a reliable method for periodic reexamination to determine the efficacy of treatment.

Although it is more operator dependent than transabdominal ultrasonography, EUS provides not only imaging capability but also adds the capacity to obtain cytologic and chemical samples of tissue and fluid aspirated with linear array monitoring.

142
Q

Role of CT scan in chronic pancreatitis?

A

CT scanning has affected the diagnosis of pancreatic disease more broadly than any other method.

Duct dilatation, calculous disease, cystic changes, inflammatory events, and anomalies are all detectable with a resolution of 3 to 4 mm (Fig. 33-28).

CT scanning has a false-negative rate of <10% for chronic pancreatitis, but early or mild chronic disease may go undetected by CT imaging.

The earliest changes are dilatation of secondary ducts and heterogeneous parenchymal changes, which are detectable by EUS and ERCP.

Another drawback of CT scanning is its lower sensitivity for detecting small neoplasms, which are seen with increased frequency in chronic pancreatitis and may be invisible to all modalities except EUS.

143
Q

Endoscopic ultrasound features of chronic pancreatitis?

A

DUCTAL CHANGES
Duct size>3mm (ductal dilatation)
Tortuous pancreatic duct (duct irregularity)
Intraductal echogenic foci (stones or calcification)
Echogenic duct wall (ductal fibrosis)
Side-branch ectasia (periductal fibrosis)

PARENCHYMAL CHANGES
Inhomogenous echopattern (edema)
Reduced echogenic foci (1-3mm) (edema)
Enhanced echogenic foci (calcifications)
Prominent inter lobular septae (fibrosis)
Lobular outer gland margin (fibrosis, glandular atrophy)
Large echo-poor cavities (>5mm) (pseudocyst)

144
Q

Role of MRI in chronic pancreatitis?

A

The resolution of cross-sectional MRI scanning is now approaching that of CT scanning, although the availability of MRI scanners and the complexity of the images produced have limited their large-scale use for routine imaging of the pancreas.

MRCP has been shown to be an effective screening technique for disclosing ductal abnormalities that correlates closely with the contrast-filled ducts imaged by ERCP.

The advantages of MRCP include its noninvasive methodology and its ability to image obstructed ducts that are not opacified by ERCP injection.

It is therefore a useful screening study to detect duct abnormalities and to confirm the need for interventional procedures.

Oral, IV, and intraductal contrast are unnecessary for MRCP, and its lack of ionizing radiation makes this the safest method to image the ductal system in high-risk patients.

145
Q

Role of ERCP in chronic pancreatitis?

A

For the diagnosis and staging of chronic pancreatitis, ERCP is considered to be the gold standard.

It also serves as a vehicle that enables other diagnostic and therapeutic maneuvers, such as biopsy or brushing for cytology, or the use of stents to relieve obstruction or drain a pseudocyst (Fig. 33-30).

Unfortunately, ERCP also carries a risk of procedure-induced pancreatitis that occurs in approximately 5% of patients.

Patients at increased risk include those with sphincter of Oddi dysfunction and those with a previous history of post-ERCP pancreatitis.

Post-ERCP pancreatitis occurs after uncomplicated procedures, as well as after those that require prolonged manipulation.

Severe pancreatitis and deaths have occurred after ERCP. It should be reserved for patients in whom the diagnosis is unclear despite the use of other imaging methods, or in whom a diagnostic or therapeutic maneuver is specifically indicated.

146
Q

Three approaches in relieving pain in chronic pancreatitis?

A

(a) reducing secretion and/or decompress the secretory compartment,
(b) resecting the focus of chronic inflammatory change, or
(c) interrupting the transmission of afferent neural impulses through neural ablative procedures.

147
Q

Peripancreatic fluid collection?

A

A collection of enzyme-rich pancreatic juice that occurs early in the course of acute pancreatitis, or that forms after a pancreatic duct leak; located in or near the pancreas; it lacks a well-organized wall of granulation or fibrous tissue

148
Q

Early pancreatic (sterile) necrosis?

A

A focal or diffuse area of nonviable pancreatic parenchyma, typically occupying >30% of the gland and containing liquefied debris and fluid

149
Q

Late pancreatic (sterile) necrosis

A

An organized collection of sterile necrotic debris and fluid with a well-defined margin or wall within the normal domain of the pancreas

150
Q

Acute pseudocyst

A

A collection of pancreatic juice enclosed within a perimeter of early granulation tissue, usually as a consequence of acute pancreatitis that has occurred within the preceding 3–4 wk

151
Q

Chronic pseudocyst?

A

A collection of pancreatic fluid surrounded by a wall of normal granulation and fibrous tissue, usually persisting for >6 wk

152
Q

Pancreatic abscess?

A

Any of the above in which gross purulence (pus) is present, with bacterial or fungal organisms documented to be present

153
Q

What is the Whipple’s Triad?

A

The triad consists of symptomatic fasting hypoglycemia, a documented serum glucose level <50 mg/dL, and relief of symptoms with the administration of glucose.

Manifested in a patient with insulinoma, the most common functional pancreatic endocrine neoplasm.

154
Q

The percentage of patients who will have an occurrence of a replaced right hepatic artery is:

A. 1–2%
B. 5–10%
C. 15–20%
D. 20–25%

A

Answer: C

In 15 to 20% of patients, the right hepatic artery will arise from the superior mesenteric artery and travel upward toward the liver along the posterior aspect of the head of the pancreas (referred to as a replaced right hepatic artery).

It is important to look for this variation on preoperative computed tomographic (CT) scans and in the operating room (OR) so the replaced hepatic artery is recognized and injury is avoided.

(See Schwartz 10th ed., p. 1345.)

155
Q

The most common complication of chronic pancreatitis is

A. Pseudocysts

B. Duct strictures and/or stones

C. Pancreatic necrosis

D. Duodenal obstruction

A

Answer: A
A chronic collection of pancreatic fluid surrounded by a non-epithelialized wall of granulation tissue and fibrosis is referred to as a pseudocyst.

Pseudocysts occur in up to 10% of patients with acute pancreatitis, and in 20 to 38% of patients with chronic pancreatitis, and thus, they comprise the most common complication of chronic pancreatitis.

(See Schwartz10th ed., pp. 1375–1376.)

156
Q

Insulinomas associated with the multiple endocrine neoplasia (MEN)1 syndrome

A. Do not usually require resection

B. Are sporadic in nature

C. Have a lower rate of recurrence

D. Are more likely to be malignant

A

Answer: B

Approximately 90% of insulinomas are sporadic, and 10% are associated with the multiple endocrine neoplasia (MEN)1 syndrome.

Insulinomas associated with the MEN1 syndrome are more likely to be multifocal and have a higher rate of recurrence.

(SeeSchwartz10thed.,p.1391.)

157
Q

A pancreatic cystic neoplasm that is <3 cm, has atypical cells present, and has a solid component

A. Requires a repeat CT scan in 3 to 6 months

B. Requires a repeat CT scan in 1 year

C. Requires continued observation

D. Requires resection

A

Answer: D

See Figure 33-1. (See Schwartz 10th ed., Figure 33-75, p. 1409.)

Detailed clinical history and pancreas protocol CT

  • -> Symptomatic, or MCN/IPMN >3cm?
  • —–> Resect.

–> Asymptomatic MCN/IPMN <3 cm,
serous cystadenoma, branch duct IPMN
——> Further evaluation.

FURTHER EVALUATION

1) <1cm
- Repeat CT in 1 year.

Repeat CT:
- <1cm: Repeat CT in 1 year.

  • ≤2 cm: Repeat CT
    q 6–12 months
- 2-3cm: 
EUS -FNA MRCP/ERCP
• Mucin present
• CEA >200 ng/mL
• Atypical cells present
• Solid component
• Dilated main pancreatic duct?
---> if yes, resect.
--> If none, repeat CT
q 3–6 months .
2) 1-3cm
EUS -FNA MRCP/ERCP
• Mucin present
• CEA >200 ng/mL
• Atypical cells present
• Solid component
• Dilated main pancreatic duct?
---> if yes, resect.
--> If none, repeat CT
q 3–6 months.
158
Q

According to Ranson criteria a 67-year-old female patient suspected of acute pancreatitis presenting to the OR with sudden onset of severe abdominal pain, a serum aspartate transaminase (AST) >250U/dL, a white blood cell (WBC) >16,000/mm3, and a blood glucose >200 mg/dL would receive a disease classification of:

A. Predicted severe
B. Predicted mild,uncomplicated
C. Predicted moderate
D. A mortality of 10%

A

Answer: A

See table 33-1. (See Schwartz 10th ed,
able 33-7, p. 1356.)

159
Q

Which of the following is the most common presenting symptom in patients with a somatostatinoma?

A. Cholelithiasis
B. Constipation
C. Hypoglycemia
D. Hypocalcemia

A

Answer: A
Because somatostatin inhibits pancreatic and biliary secretions, patients with a somatostatinoma present with gallstones due to bile stasis, diabetes due to inhibition of insulin secretion, and steatorrhea due to inhibition of pancreatic exocrine secretion and bile secretion.

Most somatostatinomas originate in the proximal pancreas or the pancreatoduodenal groove, with the ampulla and periampullary area as the most common site (60%).

The most common presentations are abdominal pain (25%), jaundice (25%), and cholelithiasis (19%).

This rare type of pancreatic endocrine tumor is diagnosed by confirming elevated serum somatostatin levels, which are usually above 10 ng/mL.

Although most reported cases of somatostatinoma involve metastatic disease, an attempt at complete excision of the tumor and cholecystectomy is warranted in fit patients.

(SeeSchwartz10thed.,p.1393.)

160
Q

The etiology associated with chronic calcific pancreatitis is

A. Hyperparathyroidism
B. Hyperlipidemia
C. Alcohol abuse
D. All of the above

A

Answer: D

This type is the largest subgroup in the classification scheme proposed by Singer and Chari, and includes patients with calcific pancreatitis of most etiologies (Table 33-2).

Although the majority of patients with calcific pancreatitis have a history of alcohol abuse, stone formation and parenchymal calcification can develop in a variety of etiologic subgroups; hereditary pancreatitis and tropical pancreatitis are particularly noteworthy for the formation of stone disease.

The clinician should therefore avoid the assumption that calcific pancreatitis confirms the diagnosis of alcohol abuse.

(See Schwartz 10th ed., able 33-11, p. 1365.)

161
Q

In patients undergoing endoscopic retrograde cholangio pancreatography (ERCP) for diagnosis and staging of chronic pancreatitis, the population most at risk of developing procedure-induced pancreatitis is

A. One with calculus disease

B. One with intraductal lesions

C. One with sphincter of Oddi dysfunction

D. One with a high percentage of parenchymal calcification

A

Answer: C

Endoscopic retrograde cholangiopancreatography (ERCP) is considered to be the gold standard or the diagnosis and staging of chronic pancreatitis.

It also serves as a vehicle that enables other diagnostic and therapeutic maneuvers, such as biopsy or brushing or cytology, or the use of stents to relieve obstruction or drain a pseudocyst (Fig. 33-2).

Unfortunately, ERCP also carries a risk of procedure-induced pancreatitis that occurs in approximately 5% of patients.

Patients at increased risk include those with sphincter of Oddi dysfunction and those with a previous history of post-ERCP pancreatitis.

Post-ERCP pancreatitis occurs after uncomplicated procedures, as well as after those that require prolonged manipulation.

Severe pancreatitis and deaths have occurred after ERCP, it should be reserved for patients in whom the diagnosis is unclear despite the use of other imaging methods, or in whom a diagnostic or therapeutic maneuver is specifically indicated.

(See Schwartz 10th ed., Figure 33-29, pp. 1370–1371.)

162
Q

Treatment of a 1-cm gastrinoma in the wall of the duodenum is best accomplished by

A. Enucleation
B. Full-thickness resection
C. Duodenectomy
D. Whipple procedure

A

Answer: B
Fifty percent of gastrinomas metastasize to lymph nodes or the liver, and are therefore considered malignant. Patients who meet criteria for operability should undergo exploration for possible removal of the tumor.

Although the tumors are submucosal, a full-thickness excision of the duodenal wall is performed if a duodenal gastrinoma is found.

All lymph nodes in Passaro triangle are excised for pathologic analysis.

If the gastrinoma is found in the pancreas and does not involve the main pancreatic duct, it is enucleated.

Pancreatic resection is justified for solitary gastrinomas with no metastases.

A highly selective vagotomy can be performed if unresectable disease is identified or if the gastrinoma cannot be localized. This may reduce the amount of expensive proton pump inhibitors required.

In cases in which hepatic metastases are identified, resection is justified if the primary gastrinoma is controlled and the metastases can be safely and completely removed.

Debulking or incomplete removal of multiple hepatic metastases is probably not helpful, especially in the setting of MEN1.

The application of new modalities such as radiofrequency ablation seems reasonable, but data to support this approach are limited.

Postoperatively, patients are followed with fasting serum gastrin levels, secretin stimulation tests, octreotide scans, and CT scans.

In patients found to have inoperable disease, chemotherapy with streptozocin, doxorubicin, and 5-fluorouracil (5-FU) is used.

Other approaches such as somatostatin analogues, interferon, and chemoembolization also have been used in gastrinoma with some success. (See Schwartz 10th ed., p. 1392.)

163
Q

The ERCP finding that is virtually diagnostic of intraductal papillary mucinous neoplasms (IPMNs) is

A. A fish-eye lesion

B. Calcification

C. Beaded or chain-of-lakes appearance of theduct

D. Cysts that resemble serous cystadenomas

A

Answer: A

Intraductal papillary mucinous neoplasms (IPMNs) usually occur within the head of the pancreas and arise within the pancreatic ducts.

The ductal epithelium forms a papillary projection into the duct, and mucin production causes intraluminal cystic dilation of the pancreatic ducts (Fig. 33-3).

Imaging studies demonstrate diffuse dilation of the pancreatic duct, and the pancreatic parenchyma is often atrophic due to chronic duct obstruction.

However, classic features of chronic pancreatitis, such as calcification and a beaded appearance of the duct, are not present.

At ERCP, mucin can be seen extruding from the ampulla of Vater, also called fish-eye lesion, that is virtually diagnostic of IPMN (Fig.33-4).

(See Schwartz 10th ed., Figures 33-77 and 33-8, pp. 1411–1412.)

164
Q

Pain from chronic pancreatitis can be caused by

A. Ductal hypertension

B. Parenchymal disease

C. Obstructive pancreatopathy

D. All of the above

A

Answer: D
Pain from chronic pancreatitis has been ascribed to three possible etiologies.

Ductal hypertension, due to strictures or stones, may predispose to pain that is initiated or exacerbated by eating.

Chronic pain without exacerbation may be related to parenchymal disease or retroperitoneal inflammation with persistent neural involvement.

Acute exacerbations of pain in the setting of chronic pain may be due to acute increases in duct pressure or recurrent episodes of acute inflammation in the setting of chronic parenchymal disease.

Nealon and Matin have described these various pain syndromes as being predictive of the response to various surgical procedures.

Pain that is found in association with ductal hypertension is most readily relieved by pancreatic duct decompression, through endoscopic stenting or surgical decompression.

(See Schwartz 10th ed., p. 1371.)

165
Q

A patient undergoing the Frey procedure to relieve pain from obstructive pancreatopathy is found to have 85% parenchymal fibrosis. The percentage of pain relief the patient is likely to experience is

A. 50%
B. 10%
C. 100%
D. 60%

A

Answer: C

The surgical relief of pain due to obstructive pancreatopathy may be dependent on the degree of underlying fibrosis rather than the presence of ductal obstruction, per se, according to a recent study from Johns Hopkins by Cooper et al.

Thirty-five patients with chronic pain associated with evidence of duct obstruction were treated with local resection of the pancreatic head and longitudinal pancreatico-jejunostomy (LR-LPJ), or Frey procedure, and the degree of pain resolution after surgery was compared to the degree of underlying parenchymal fibrosis.

After a follow-up that averaged 22 months, patients with more than 80% fibrosis had 100% pain relief, whereas only 60% patients with less than 10% fibrosis experienced substantial or complete pain relief (Fig. 33-5).

These findings suggest that minimal fibrosis, or “minimal change chronic pancreatitis,” may produce chronic pain due to extrapancreatic or “peri-pancreatic” inflammatory events which are not ameliorated by decompression.

(See Schwartz 10th ed., Figure 33-31, pp. 1371–1372.)

166
Q

The only therapy shown to prevent the progression of chronic pancreatitis is

A. Pancreatic duct decompression

B. Major resection

C. Transduodenal sphincteroplasty

D. Roux-en-Y pancreaticojejunostomy

A

Answer: A

The traditional approach to surgical treatment of chronic pancreatitis and its complications has maintained that surgery should be considered only when the medical therapy of symptoms has ailed.

Nealon and Thompson published a landmark study in 1993, however, that showed that the progression of chronic obstructive pancreatitis could be delayed or prevented by pancreatic duct decompression.

No other therapy has been shown to prevent the progression of chronic pancreatitis, and this study demonstrated the role of surgery in the early management of the disease (Table 33-3).

Small-duct disease or “minimal change chronic pancreatitis” are causes or uncertainty over the choice of operation, however.

Major resections have a high complication rate, both early and late, in chronic alcoholic pancreatitis, and lesser procedures often result in symptomatic recurrence.

So the choice of operation and the timing of surgery are based on each patient’s pancreatic anatomy, the likelihood (or lack thereof) that further medical and endoscopic therapy will halt the symptoms of the disease, and the chance that a good result will be obtained with the lowest risk of morbidity and mortality.

Finally, preparation for surgery should include restoration of protein-caloric homeostasis, abstinence from alcohol and tobacco, and a detailed review of the risks and likely outcomes to establish a bond of trust and commitment between the patient and the surgeon. (See Schwartz 10th ed., able 33-20, p. 1382.)

167
Q

The part of the pancreas resected in order to ensure successful resolution of pain long-term for patients with chronic pancreatitis is

A. The head

B. The body

C. The neck

D. The tail

A

Answer: A

The common element of these variations on the theme of LR-LPJ remains the excavation or “coring out” of the central portion of the pancreatic head.

It remains uncertain, however, whether and to what degree the dichotomy needs to be extended into the body and tail.

The logical conclusion of all of these efforts is that the head of the pancreas is the nidus of the chronic inflammatory process in chronic pancreatitis, and that removal of the central portion of the head of the gland is the key to the successful resolution of pain long-term. (See Schwartz 10th ed., pp. 1342 and 1387.)

168
Q

In pylorus-preserving resections of the pancreas, the technique with the lowest rate of pancreatic leakage is

A. Stent
B. Glue
C. Octreotide
D. All of the above

A

Answer: D
The preservation of the pylorus has several theoretical advantages, including prevention of reflux of pancreaticobiliary secretions into the stomach, decreased incidence of marginal ulceration, normal gastric acid secretion and hormone release, and improved gastric function.

Patients with pylorus- preserving resections have appeared to regain weight better than historic controls in some studies. Return of gastric emptying in the immediate postoperative period may take longer after the pylorus-preserving operation, and it is controversial whether there is any significant improvement in long-term quality of life with pyloric preservation.

Techniques for the pancreaticojejunostomy include end- to-side or end-to-end and duct-to-mucosa sutures or invagination (Fig. 33-6).

Pancreaticogastrostomy has also been investigated.

Some surgeons use stents, glue to seal the anastomosis, or octreotide to decrease pancreatic secretions.

No matter what combination of these techniques is used, the pancreatic leakage rate is always about 10%.

Therefore, the choice of techniques depends more on the surgeon’s personal experience.

(See Schwartz 10th ed., Figure 33-73, p. 1403.)

169
Q

The prognosis factor that does NOT decrease survival rates in patients with gastrinomas is

A. Liver metastases
B. Absence of MEN 1
C. Lymph node metastases
D. Primary tumor located outside of Passaro triangle

A

Answer: C

Fifty percent of gastrinomas metastasize to lymph nodes for the liver, and are therefore considered malignant.

Patients who meet criteria for operability should undergo exploration for possible removal of the tumor.

Although the tumors are submucosal, a full-thickness excision of the duodenal wall is performed if a duodenal gastrinoma is found.

All lymph nodes in Passaro triangle are excised for pathologic analysis.

If the gastrinoma is found in the pancreas and does not involve the main pancreatic duct, it is enucleated.

Pancreatic resection is justified for solitary gastrinomas with no metastases.

A highly selective vagotomy can be performed if unresectable disease is identified or if the gastrinoma cannot be localized. This may reduce the amount of expensive proton pump inhibitors required.

In cases in which hepatic metastases are identified, resection is justified if the primary gastrinoma is controlled and the metastases can be safely and completely removed.

Debulking or incomplete removal of multiple hepatic metastases is probably not helpful, especially in the setting of MEN1.

The application of new modalities such as radiofrequency ablation seems reasonable, but data to support this approach are limited.

Postoperatively, patients are followed with fasting serum gastrin levels, secretin stimulation tests, octreotide scans, and CT scans. In patients found to have inoperable disease, chemotherapy with streptozocin, doxorubicin, and 5-fluorouracil (5-FU) is used.

Other approaches such as somatostatin analogues, interferon, and chemoembolization also have been used in gastrinoma with some success.

Unfortunately, a biochemical cure is achieved in only about one-third of the patients operated on for Zollinger-Ellison syndrome (ZES).

Despite the lack of success, long-term survival rates are good, even in patients with liver metastases.

The 15-year survival rates for patients without liver metastases is about 80%, while the 5-year survival rate for patients with liver metastases is 20 to 50%.

Pancreatic tumors are usually larger than tumors arising in the duodenum, and more often have lymph node metastases.

In gastrinomas, liver metastases decrease survival rates, but lymph node metastases do not. The best results are seen after complete excision of small sporadic tumors originating in the duodenum.

Large tumors associated with liver metastases, located outside of Passaro triangle, have the worst prognosis.

(See Schwartz 10th ed., p. 1392.)

170
Q

A 58-year-old woman with jaundice underwent endoscopic retrograde cholangiopancreatography (ERCP) (see Fig. 26.1) as a part of her diagnostic workup. On the basis of this radiograph, what is the most likely diagnosis?

A. Chronic pancreatitis 
B. Pancreatic cancer
C. Cholangiocarcinoma 
D. Pancreas divisum
E. Ectopic pancreas
A

ANSWER: B

COMMENTS: This ERCP study shows the classic “double-duct sign:” dilation of the biliary system above an area of abrupt narrowing and abrupt termination of the main pancreatic duct.

These findings place the primary abnormality in the geographic location of the pancreatic head, and it is not uncommon for a pancreatic neoplasm to involve both ducts.

Chronic pancreatitis may cause biliary obstruction, but the obstruction in the biliary systems is usually more distal.

Likewise, there are no coexistent changes in this patient, such as irregular beading of the pancreatic duct, to suggest that chronic pancreatitis is present.

Cholangiocarcinoma may be responsible for the stenosis in the biliary system, but cholangiocarcinomas rarely become large enough to involve the pancreatic duct.

With pancreas divisum, injection of the major papilla opacifies only a short, tapering ventral duct draining the caudal portion of the pancreatic head and uncinate process.

Injection of the minor papilla demonstrates the dorsal duct draining the major portion of the gland.

171
Q

You diagnose the patient in Question 46 with an insulinoma. Magnetic resonance imaging (MRI) shows a lesion in the body of the pancreas. Which of the following statements is true regarding the preferred treatment?

A. Diazoxide is the preferred initial method of management.

B. Enucleation is acceptable for localized pancreatic lesions.

C. Because most lesions are multiple or diffuse, total or nearly total pancreatectomy is generally necessary.

D. Because most lesions are malignant, adjuvant streptozocin is usually indicated.

E. Parathyroid adenoma should be excluded or treated before pancreatic resection.

A

ANSWER: B

COMMENTS: Insulinomas are usually single and benign and are rarely ectopic.

Localization of an insulinoma can be difficult, and preoperative imaging along with thorough mobilization and exploration of the pancreas is mandatory.

Intraoperative ultrasonography is indispensable.

For localized lesions, simple enucleation is the pre- ferred treatment, but the integrity of the pancreatic duct must be ascertained.

If the lesion cannot be identified and the biochemical basis of the diagnosis is firm, blind distal pancreatic resection with a careful histologic examination of the specimen may be necessary.

Intraoperative monitoring of serum glucose levels has also been used.

Diazoxide inhibits insulin release from beta cells and is occasionally used for preoperative control or for patients with recurrent postoperative hypoglycemia.

For patients with metastatic malignant insulinoma, tumor debulking may be beneficial, as is the use of streptozocin and 5-fluorouracil.

Gastrinoma, not insulinoma, is the most common pancreatic adenoma associated with multiple endocrine adenomatosis type I syndrome.

Parathyroid disease should be excluded or treated before surgical intervention for gastrinoma.

172
Q

Which of the following features is characteristic of Zollinger-
Ellison syndrome but not of Verner-Morrison syndrome?

A. Diarrhea

B. Hypercalcemia

C. Hypocalcemia

D. Increased gastric acid secretion

E. Malignancy

A

ANSWER: D

COMMENTS: Zollinger-Ellison syndrome is caused by a gastrin- producing islet cell tumor. Verner-Morrison syndrome is caused by an islet cell tumor that produces vasoactive intestinal peptide. Zollinger- Ellison syndrome is associated with a marked increase in gastric acid secretion and with diarrhea.

Hypercalcemia may occur because of associated parathyroid abnormalities.

Verner-Morrison syndrome is characterized by watery diarrhea, hypokalemia, and achlorhydria.
Hypercalcemia may occur, but the parathyroids are usually normal.

Both syndromes are frequently the result of malignant islet cell tumors.

173
Q

Which of the following is not a feature of the clinical syndrome associated with a glucagon-producing islet cell tumor?

A. Rash

B. Diabetes

C. Seizures

D. Glossitis

E. Anemia

A

ANSWER: C

COMMENTS: Patients with glucagon-secreting tumors have diabetes, anemia, weight loss, venous thrombosis, glossitis, and a characteristic cutaneous lesion known as necrolytic migratory erythema.

The lesion is rare and often metastatic at the time of diagnosis.

Treatment is directed at achieving as complete a resection as possible.

Postoperatively, chemotherapy with dacarbazine or streptozocin may be useful for a residual or recurrent disease.

174
Q

A 27-year-old male presents to the trauma bay after being hit in the mid-abdomen with a large steel post at the construction site where he works. Focused assessment with sonography for trauma scan of the abdomen shows free fluid, and the patient is becoming more tachycardic in the trauma bay despite fluids and pain control. He is taken to the operating room, and complete transection of the pancreatic neck is identified. There are no associated organ injuries. Which of the following treatments is most appropriate?

A. Placement of drains and closure of the abdomen

B. Distal pancreatectomy with ligation of the proximal duct

C. Roux-en-Y pancreaticojejunostomy to the distal pancreas with ligation of the proximal duct

D. Roux-en-Y pancreaticojejunostomy to both the proximal and distal segments of the pancreas

E. Pancreaticoduodenectomy with Roux-en-Y pancreaticojejunostomy to the distal duct

A

ANSWER: B

COMMENTS: Pancreatic contusions or lacerations without ductal disruption are managed by drainage alone.

The pancreatic neck is a frequent site of pancreatic injury when it occurs with blunt trauma.

Distal pancreatectomy with identification and closure of the proximal duct and drainage is safe, and resections involving up to 80% of an otherwise normal gland can be accomplished without subsequent endocrine insufficiency.

In theory, Roux-en-Y pancreaticojejunostomy may be desirable to preserve pancreatic tissue, but it is not recommended for the management of acute injuries because of the risk associated with a pancreatic anastomosis and the need to open the gut.

Pancreaticoduodenectomy is indicated for patients with severe combined duodenal, pancreatic, and bile duct injuries.

Most pancreatic injuries are the result of penetrating trauma, although the gland is vulnerable to blunt trauma because of its fixed position anteriorly over the vertebral column.

The presence of significant pancreatic injury following blunt trauma is often not initially apparent.

Hyperamylasemia in serum or peritoneal fluid suggests the diagnosis, but a negative peritoneal tap or lavage does not exclude retroperitoneal injury.

Retroperitoneal hematomas in the upper part of the abdomen should be explored to exclude pancreatic ductal injury.

Pancreatitis is the most common cause of pseudocyst, although about 25% occur as a result of trauma.

175
Q

The mechanism of alcohol-induced acute pancreatitis is
thought to involve all of the following except:

A. Pancreatic ductal obstruction

B. Pancreatic exocrine hypersecretion

C. Hypertriglyceridemia

D. Acetaldehyde toxicity

E. Genetic defect in lysosomal membranes

A

ANSWER: E

COMMENTS: Ethanol is the prevalent etiologic factor in acute pancreatitis.

There are several contributory mechanisms by which alcohol-induced pancreatic injury occurs.

Ethanol causes pancreatic ductal hypertension by increasing ampullary resistance and by intraductal deposition of stone proteins.

Concomitantly, ethanol stimulates gastric acid secretion and increases pancreatic exocrine secretion via the release of secretin.

The combination of ductal obstruction with stimulated secretion may result in enzyme extravasation.

Acetaldehyde, the metabolic product of ethanol, injures acinar cells by increasing membrane permeability and disrupting the microtubule structure.

The elevated levels of serum triglycerides induced by alcohol are a source of cytotoxic free fatty acids.

Alcohol also impairs normal trypsin inhibition and reduces pancreatic blood flow.

All of these effects may contribute to intraglandular enzyme activation and the development of acute alcoholic pancreatitis.

176
Q

In North America, chronic pancreatitis is most commonly related to chronic alcohol ingestion. Which of the following is the second most common cause?

A. Gallstones 
B. Drugs
C. Infection
D. Malnutrition 
E. Idiopathic
A

ANSWER: E
COMMENTS: In the Western world, alcohol use accounts for about 75% of cases of chronic pancreatitis.

Approximately 20% of cases are considered idiopathic. In parts of Africa and Asia, protein malnutrition is an important etiologic factor.

Other, less common causes of chronic pancreatitis include pancreatic duct obstruction (secondary to stenosis or pancreas divisum), hyperparathyroidism, trauma, cystic fibrosis (CF), and hereditary causes.

Unlike acute pancreatitis, calculous biliary disease is not a typical cause of chronic pancreatitis.

Certain infections (particularly viral) and drugs are among the many factors that can produce acute, rather than chronic, pancreatitis.

177
Q

Endoscopy demonstrates a 1-cm submucosal nodule with central umbilication in the second portion of the duodenum. This finding is usually associated with which of the following?

A. Peptic ulceration

B. Increased risk for pancreatic cancer

C. Islet cell hyperplasia

D. Absence of symptoms

E. Intussusception

A

ANSWER: D

COMMENTS: A heterotopic pancreas is a pancreatic tissue located at sites other than the normal location of the gland.

Ectopic pancreatic tissue has been described at many anatomic locations but is typically found in the stomach, the duodenum, or a Meckel’s diverticulum.

Theories of origin include metaplasia (the favored theory) and transplantation. Histologic findings range from those of a rudimentary structure to a fully formed gland.

Most heterotopic rests contain ducts, and both endocrine and exocrine elements may be present.

This entity is not uncommon, being described in 1%–2% of autopsies. It is usually asymptomatic. When symptoms occur, they are related to the location of the ectopic site and include obstruction (as a result of intussusception), ulceration, and bleeding.

Although malignancy has been reported, there is no evidence that heterotopic pancreatic tissue is predisposed to cancer.

The typical gross appearance is a submucosal nodule, often with central umbilication.

Resection is indicated for symptomatic lesions and is appropriate diagnostically for incidental lesions discovered during operations for other reasons.

178
Q

What is the recommended treatment of duodenal obstruction
caused by an annular pancreas?

A. Endoscopic division of the associated duodenal web

B. Gastrojejunostomy

C. Duodenoduodenostomy

D. Surgical division of the annular tissue

E. Pancreaticoduodenectomy

A

ANSWER: C

COMMENTS: An annular pancreas is a congenital anomaly involving a band of pancreatic tissue encircling the second portion of the duodenum.

The annular tissue appears to originate from the embryologic ventral pancreas.

Causal theories include abnormal fixation of the ventral pancreatic primordium before gut rotation, failure of involution of part of the ventral pancreas, and the development of heterotopic pancreatic tissue in the duodenum.

Approximately one-half of these cases are diagnosed in infants and the remainder in adults, with a peak during the fourth decade of life.

Most patients are asymptomatic.

Clinical findings are obstruction in infants and children and obstruction, ulceration, or pancreatitis in adults.

Associated anomalies include duodenal stenosis or atresia and Down syndrome.

Treatment of symptomatic patients consists of surgical bypass by duodenoduodenostomy or duodenojejunostomy.

Gastrojejunostomy can also alleviate obstruction but risks marginal ulceration.

Resection or division of the annular band is not advised because it risks the development of a pancreatic fistula and may fail to relieve the obstruction.

179
Q

The embryologic ventral pancreas forms which area of the fully developed gland?

A. Superior head

B. Uncinate process

C. Neck

D. Body

E. None of the above because it regresses

A

ANSWER: B

COMMENTS: The pancreas is formed from two outpouchings of the primitive gut.

The dorsal pancreas originates from the duodenum, and the ventral pancreas begins as a bud from the hepatic diverticulum, which itself is an outpouching of the duodenum.

Other outgrowths from the hepatic diverticulum mature into the liver, gallbladder, and bile ducts.

During normal fetal development, the ventral pancreas rotates along with the primitive gut and fuses with the dorsal component.

The ventral pancreas constitutes the uncinate process and the inferior portion of the head of the gland in the fully developed state, and the dorsal pancreas forms the remainder of the gland.

Abnormalities in this developmental process result in recognized congenital anomalies that can be clinically important.

An understanding of this embryologic development is also important for recognizing the relationship of the pancreas to adjacent vascular structures during pancreatic operations.

180
Q

A 68-year-old man presents with painless jaundice. He has a history of diabetes, hypertension, and alcohol abuse. He has a 40-pack-year tobacco history. He has been hospitalized for pancreatitis three times in the past. Computed tomography (CT) scan shows a 2-cm lesion in the head of the pancreas. Endoscopic ultrasound (EUS) and biopsy confirm pancreatic adenocarcinoma. He asks you what most likely would have contributed to this diagnosis of ductal adenocarcinoma of the pancreas.

A. Chronic pancreatitis

B. Diabetes mellitus

C. Cigarette smoking

D. Coffee consumption

E. Alcohol consumption

A

ANSWER: C

COMMENTS: Epidemiologic studies have identified various demographic, medical, environmental, and dietary factors that have some relationship to pancreatic cancer.

The most firmly established risk factor is cigarette smoking. Experimentally, nitrosamines have been found to be carcinogenic. In addition, the carcinogens in cigarettes have been related to K-ras oncogene mutations, which are frequent in pancreatic cancer.

Alcohol has not been demonstrated conclusively to be a risk factor independent of cigarettes.

The previously reported association of pancreatic cancer with coffee consumption is questionable.

Diets high in fats and meat may be associated with pancreatic cancer, whereas diets high in fruits and vegetables may be protective.

Certain occupational and industrial exposures have an increased risk.

There may be some association with diabetes mellitus and certain forms of chronic pancreatitis, but the relationship is not considered causal.

Previous gastrectomy has been associated with increased risk, whereas tonsillectomy has been observed to be protective.

181
Q

The medical intensive care unit service consults you on hospital day 2 for a 57-year-old male being treated for alcoholic pancreatitis. At admission, his white blood cell (WBC) count was 17,000, glucose was 100 mg/dL, aspartate aminotransferase (AST) was 400 IU/L, alanine aminotransferase (ALT) was 150 IU/L, and lactate dehydrogenase (LDH) was 200 IU/L. Today his creatinine is 2.1 mg/dL with a stable blood urea nitrogen (BUN), his base deficit is 7 mEq/L, pO2 is 72 mmHg, calcium is 7.5 mg/dL, and base deficit is 8 mEq/L. He does not appear to be heavily third spaced. What is his likelihood of mortality?

A. 0% 
B. 10% 
C. 20% 
D. 40% 
E. 80%
A

ANSWER: D

COMMENTS: Several systems have been devised to gauge the severity of acute pancreatitis. These systems involve multiple clinical, biochemical, and, sometimes, radiologic criteria.

The most widely used system in the United States, developed by Ranson, was based on retrospective analysis and subsequent prospective verification.

The Ranson criteria include 11 parameters determined at the time of admission or during the subsequent 48 h.

Patients with three or more criteria have more severe disease and are at increased risk for septic complications and death.

The criteria reflect the patient’s underlying status, the severity of the retroperitoneal inflammatory process, and the effects on renal and respiratory function.

The Ranson criteria were originally developed for alcoholic pancreatitis and have been modified somewhat for gallstone pancreatitis.

For example, a rise in the serum BUN level of more than 2 mg/dL is 1 of the 10 criteria for gallstone pancreatitis, but the rise must be more than 5 mg/dL to meet the criteria for alcoholic pancreatitis (a subtle point).

The criteria include the following at admission: 
age > 55 years, 
WBC > 16 cells per MCL, 
blood glucose > 200 mg/dL, 
AST > 250 IU/L, and 
LDH > 350 IU/L. 
At 48 h from admission, the additional criteria include the following: 
calcium < 8 mg/dL, 
hematocrit fall > 10%, 
hypoxemia (pO2 < 60 mmHg), 
BUN increased by 5 mg/dL or more, 
base deficit > 4, and 
sequestration of fluid > 6 L. 

A score of 0 to 2 assigns a 2% mortality,
3 to 4: 15%,
5 to 6: 40%, and
greater than 7: 100%.

Other physiologic scoring systems, such as the Acute Physiology, Age, and Chronic Health Evaluation II, are also useful prognostically, although they are not designed specifically for acute pancreatitis.

182
Q

Hyperamylasemia is diagnostic of acute pancreatitis when associated with which of the following laboratory findings?

A. Hyperlipasemia

B. Increased urinary amylase levels

C. Amylase-creatinine clearance ratio (ACCR) greater than 5%

D. Hypocalcemia

E. None of the above

A

ANSWER: E
COMMENTS: The diagnosis of acute pancreatitis is based on signs and symptoms, supported by biochemical findings and morphologic abnormalities seen on imaging studies such as CT.

No biochemical feature is pathognomonic of acute pancreatitis.

Hyperamylasemia, hyperlipasemia, and elevations in urinary amylase levels and the ACCR are typical of acute pancreatitis but are not specific or sensitive, and they can occur with other abdominal and extraabdominal disorders.

Hypocalcemia may occur as a consequence of pancreatitis, but it is also nonspecific.

There is no absolute level of serum amylase or lipase that is diagnostic of acute pancreatitis.

Marked elevations are more indicative of pancreatitis but are not themselves diagnostic.

Both amylase and lipase levels may be elevated in a number of conditions that can be confused with acute pancreatitis, such as acute cholecystitis, perforated peptic ulcer, and intestinal infarction.

Moreover, severe pancreatitis can occur without substantial elevations in these serum enzymes.

Elevations in serum and urinary amylase levels and in the ACCR, as determined by the following equation, are typical of acute pancreatitis.

ACCR = Uamy/Samy × Scr/Ucr ×100

where U = urine, S = serum, amy = amylase, and cr = creatinine.

Elevation of the ACCR above the normal 2%–5% range is not specific for pancreatitis, but a normal ratio in the presence of hyperamylasemia suggests that the hyperamylasemia is the result of something other than pancreatitis.

Serum and urinary amylase levels and the ACCR may be normal in patients with chronic pancreatitis or elevated during an acute exacerbation.

Renal disease may be associated with low urinary amylase levels and an elevated ACCR.

Common duct stones may produce hyperamylasemia without true pancreatitis.

The urinary amylase level is elevated, although the ACCR may be normal.

With macroamylasemia, amylase forms complexes with serum proteins too large for glomerular filtration.

The serum amylase level is therefore elevated, but urinary amylase levels and the ACCR are low.

The diagnosis can be confirmed by electrophoresis.

Abdominal pain has been reported in more than one-half of patients with macroamylasemia, although the biochemical abnormality is probably not etiologically related to the pain.

Hyperamylasemia predominantly caused by salivary amylase may also be associated with a low urinary amylase level and ACCR because compared with the pancreatic isoenzyme, the salivary isoenzyme is cleared more slowly by the kidneys.

183
Q

At the time of laparotomy for a patient presenting with a known diagnosis of pancreatic cancer, the patient is found to have an unresectable pancreatic cancer locally invading the antrum of the stomach and retroperitoneum.
There are no lesions identified elsewhere in the abdomen. Which of the following statements regarding management is correct?

A. The preferred management is to close the patient and place an endoscopic stent postoperatively.

B. Proceed with pancreaticoduodenectomy.

C. Choledochoduodenostomy is preferred.

D. Gastrojejunostomy should be performed.

E. Gastrojejunostomy should not be performed.

A

ANSWER: D

COMMENTS: Most patients with pancreatic cancer do not have resectable disease.

Palliative treatment is directed to relieve obstruction of the bile duct and duodenum and to alleviate pain.

For lesions demonstrated to be unresectable before laparotomy, nonoperative relief of biliary obstruction can be achieved by the endoscopic (preferred) or transhepatic route.

Surgical bypass generally provides more durable relief with less need for further intervention. It is preferred for patients when unresectability is determined at the time of laparotomy.

Pancreatic cancers can obstruct the duodenum or the proximal jejunum near the ligament of Treitz.

Traditionally, many surgeons have favored routine “double bypass” (biliary and duodenal) for operated patients because the rate of duodenal obstruction that develops later in patients treated by biliary bypass alone has been cited to be 5%–30%.

However, duodenal obstruction does not develop in most patients, and gastrojejunostomy is sometimes associated with problems such as bleeding or delayed gastric emptying.

The selective approach is therefore appropriate.

Patients with obstructive symptoms or impending obstruction as a result of tumor location should undergo gastrojejunostomy.

Gastrojejunostomy is also advisable for patients with an anticipated longer survival, such as those whose lesions are not resected because of local tumor invasion but because of hepatic or peritoneal metastases.

Endoscopic placement of a duodenal stent is another option, but the results are not always satisfactory.

Additionally, choledochojejunostomy, cholecystojejunostomy, hepaticojejunostomy, and choledochoduodenostomy are each appropriate for the management of distal bile duct obstruction.

A Roux-en-Y configuration is preferred by many surgeons for choledochojejunostomy reconstruction, although a simple loop (with or without distal enteroenterostomy) also suffices.

Cholecystojejunostomy is relatively simple but should be avoided if the gallbladder is diseased or when cystic duct patency cannot be demonstrated or may be jeopardized by tumor proximity.

It is sometimes taught that choledochoduodenostomy should be avoided with malignant obstruction because of possible tumor growth and eventual reobstruction.

In reality, choledochoduodenostomy can be an effective solution provided that the common bile duct is sufficiently dilated and the duodenum is pliable and unobstructed.

184
Q

A 45-year-old woman was seen at an outside hospital with abdominal pain, and a CT scan was performed showing a pancreatic cyst. The interventional radiologist drained the cyst during that admission, and the patient was sent home. She presents now to your hospital, and a CT scan again shows a septated, 10-cm cystic mass in the head of the pancreas. Which of the following statements constitutes appropriate advice?

A. The lesion is benign and requires no intervention.

B. The lesion is malignant and probably incurable.

C. Pancreaticoduodenectomy is indicated.

D. Percutaneous needle biopsy is indicated.

E. Drainage by Roux-en-Y cyst jejunostomy is indicated.

A

ANSWER: C

COMMENTS: Cystadenoma and cystadenocarcinoma are cystic neoplasms of the pancreas that are most commonly manifested as mass lesions in middle-aged women.

Serous and mucinous types are recognized, and the risk for malignancy is significant with the mucinous variety.

Cystadenoma is more common than its malignant counterpart, but malignant transformation may occur.

EUS with sampling for cytology, mucin, and carcinoembryonic antigen can be useful for gauging the likelihood of cancer.

Without resection, however, exclusion of malignancy can be difficult.

Internal drainage of cystic neoplasms is not an appropriate therapy.

Complete excision should be carried out whenever possible.

The 5-year survival rate after resection of cystadenocarcinoma is approximately 50%.

Occasionally, islet cell tumors, ductal adenocarcinomas, or other unusual tumors (e.g., papillary and cystic pancreatic neoplasms) have cystic components.

185
Q

The uncinate process of the pancreas is directly adjacent and ventral to which of the following?

A. Splenic vein

B. Inferior vena cava

C. Superior mesenteric artery

D. Left renal vein

E. Fourth portion of the duodenum

A

ANSWER: B

COMMENTS: The pancreas can be divided into various parts: head, uncinate, neck, body, and tail.

The uncinate process is the portion of the gland that extends to the left, dorsal to the portal vein and superior mesenteric artery and ventral to the aorta and inferior vena cava.

The uncinate process is located caudad and ventral to the left renal vein and cephalad to the distal duodenum.

Understanding the extent and location of the uncinate is important during resection of the head of the pancreas.

The blood supply of the uncinate is derived from numerous short branches of the superior mesenteric artery and portal vein.

When performing pancreatico- duodenectomy, these branches must be carefully controlled to prevent bleeding and avoid injury to the superior mesenteric artery or portal vein.

186
Q

Which of the following vascular relationships is not an important consideration during resection of the head of the pancreas?

A. Arterial supply of the pancreatic head from the splenic artery

B. Confluence of the splenic vein and superior mesenteric vein dorsal to the pancreatic neck

C. Absence of ventral portal vein branches dorsal to the pancreatic neck

D. Origin of the right hepatic artery from the superior mesenteric artery

E. Origin of the middle colic artery from the superior mesenteric artery

A

ANSWER: A

COMMENTS: The relationship of the pancreas to neighboring organs and to critical vascular structures is of great surgical significance.

The arterial supply to the head of the gland is derived from both the gastroduodenal and the superior mesenteric arteries via the anterior and posterior pancreaticoduodenal arcades.

For the most part, the head of the pancreas and the duodenum have a shared blood supply, so they must generally be resected together.

However, techniques for “duodenal-sparing” resection of the pancreatic head or “pancreatic-sparing” duodenectomy are appropriate in selected circumstances.

The body and tail of the pancreas receive their blood supply mainly from multiple branches of the splenic artery, which also connect with superior mesenteric sources.

Variations in major arteries—such as the origin of the right hepatic artery from the superior mesenteric artery and the origin of the middle colic artery from the superior mesenteric artery or dorsal pancreatic artery—place these vessels in close proximity to the head and neck of the pancreas, where they are subject to injury during pancreatectomy.

The junction of the splenic vein and superior mesenteric vein to form the portal vein lies behind the neck of the pancreas.

Usually, these vessels do not have large anterior tributaries in this area, but appropriate caution must nonetheless be exercised when developing this plane during pancreatic operations.

187
Q

A jaundiced, otherwise healthy patient is noted to have a 3-cm mass in the head of the pancreas on CT. EUS-guided fine-needle aspiration shows cancer. The mass abuts the portal vein, but there is no clear evidence of vessel involvement or metastatic disease. You proceed to the operating room. Intraoperatively, you are concerned about the resectability of the tumor. In which of the following situations is resection contraindicated?

A. 70% abutment of the superior mesenteric artery

B. Tumor located in the neck of the pancreas

C. Inability to verify malignancy histologically before resection

D. 40% abutment of the superior mesenteric vein

E. Tumor invading the portal vein

A

ANSWER: A

COMMENTS: When the clinical situation suggests a resectable pancreatic neoplasm in a good-risk patient with biliary obstruction, surgery for potential resection is generally indicated without additional tests.

Routine preoperative biliary decompression is not advantageous in this setting because it does not improve operative outcomes and may increase the morbidity associated with resection.

Endoscopic biliary decompression is invaluable, of course, for palliation of obstruction in patients deemed inoperable or if operation is to be delayed.

CT or MRI is usually adequate in assessing the extent of disease and resectability preoperatively.

EUS is extremely useful for identifying small tumors that are inapparent on CT, for obtaining cytologic material, and somewhat for assessing vascular invasion.

Imaging criteria for what constitutes a borderline case vary but would commonly include tumors that abut a substantial (half or greater) circumference of adjacent vessels (hepatic artery, superior mesenteric artery, portal vein) or that narrow the portal-splenic vein confluence.

Resection of a pancreatic malignancy offers the only chance for cure.

Most commonly, resection of ductal carcinomas involves pancreaticoduodenectomy because most potentially resectable tumors are located in the head or uncinate process of the gland.

Tumors originating in the body or tail of the pancreas are often not diagnosed until they are beyond the confines of surgical resection.

However, location alone does not contraindicate resection because, stage for stage, tumors in the body have the same survival as tumors in the head of the pancreas.

Resection is indicated for physiologically fit patients (age alone is not a contraindication) who do not have metastases beyond the field of resection.

Histologic or cytologic confirmation of malignancy can often be obtained intraoperatively but is not necessary before resection if the clinical circumstances suggest cancer and the surgeon is appropriately experienced.

For some tumors with local vascular invasion, en bloc resection with reconstruction of the involved vessels is appropriate if a tumor-free resection can be accomplished.

Positive lymph nodes outside the resection field, peritoneal metastases, and liver metastases generally contraindicate resection for adenocarcinoma of the exocrine pancreas.

However, tumor “debulking” and resection of liver metastases can be beneficial in patients with functioning tumors of the endocrine pancreas.

188
Q

Which of the following would not be appropriate for the management of steatorrhea in a patient with chronic pancreatitis?

A. Restriction of fat to 75 g/day

B. Encapsulated pancreatic enzymes

C. Encapsulated pancreatic enzymes and a proton pump inhibitor

D. Nonencapsulated pancreatic enzymes

E. Nonencapsulated pancreatic enzymes and a proton pump inhibitor

A

ANSWER: C

COMMENTS: Gross steatorrhea and diarrhea occur when pancreatic exocrine function is reduced to about 10% of normal.

Therapy involves limitation of fat intake and administration of adequate amounts of exogenous pancreatic enzyme preparations to provide at least 10% of the normal lipolytic activity in the duodenum at the time when the food substrate is present.

Various commercial formulations of pancreatic enzymes are available.

Nonencapsulated forms may improve the malabsorption but can be ineffective because of inactivation in the stomach when the pH falls below 4.

The addition of H2 blockers may then be useful.

Enteric-coated preparations release their enzymes at a pH above 5. Therefore they are useful for patients whose gastric pH remains low to ensure that the enzyme is not released until it reaches the duodenum.

The use of encapsulated forms with H2 blockers is counter- productive because the enzyme is released in the stomach and is then inactivated if the pH falls.

In addition, enteric-coated preparations are microspheres of varying sizes, and the larger ones do not empty into the duodenum until after the food substrate does.

189
Q

A 42-year-old female presents to the emergency department (ED) with nausea, vomiting, and pain in the epigastrium and right upper quadrant (RUQ). She is afebrile and hemodynamically stable. Her WBC is 10,000 cells per MCL without a left shift, and total bilirubin is 2.5 mg/dL. The common bile duct was not identified on subsequent ultrasound due to overlying bowel gas. Magnetic resonance cholangiopancreatography (MRCP) confirms a gallstone in the common bile duct with proximal dilation. Incidentally, she is found to have pancreatic divisum. What is the etiology of this?

A. Aplasia of the dorsal pancreatic anlage

B. Aplasia of the ventral pancreatic anlage

C. Incomplete rotation of the ventral pancreatic anlage

D. Failed fusion of the ventral and dorsal pancreatic parenchyma

E. Failed fusion of the ventral and dorsal pancreatic ducts

A

ANSWER: E

COMMENTS: Pancreas divisum currently refers to congenital variations of the pancreatic ducts that result from failed or incomplete fusion of the embryologic ventral and dorsal ductal systems.

(Historically, the term may also refer to the rare failure of parenchymal fusion.) There may be complete separation of the ducts, an absent or minimal ventral duct, or only a few meager connections between the systems.

As a consequence, most of the pancreatic duct drainage is through the dorsal duct joining the duodenum at the minor papilla.

Any existing ventral ducts (Wirsung’s) drain only the uncinate process and the caudal head of the gland rather than the bulk of the gland at the major papilla, as when normally developed.

Some variation of pancreas divisum is present in about 10% of the population. In some individuals, it is clinically significant if the relatively stenotic minor papilla imposes an obstruction to ductal flow.

This can potentially result in recurrent abdominal pain, acute pancreatitis, or even chronic pancreatitis.

The diagnosis is usually made by ERCP, and cannulation of the minor papilla may be required to image the dorsal duct. MRCP might also demonstrate this ductal anatomy.

The vast majority (95%) of individuals with pancreas divisum are asymptomatic.

190
Q

The bicarbonate concentration of pancreatic secretions is:

A. Primarily increased by cholecystokinin (CCK)

B. Primarily decreased by secretin

C. Independent of acinar cell secretion

D. Reciprocally related to the chloride concentration

E. Reciprocally related to the sodium concentration

A

ANSWER: D

COMMENTS: The centroacinar cells secrete a bicarbonate-rich solution by an active transport mechanism, primarily in response to secretin.

Cholecystokinin is the primary stimulant of enzyme secretion from the acinar cells.

The bicarbonate and chloride contents of pancreatic juice are reciprocally related.

As ductal flow rates increase, the bicarbonate concentration increases and the chloride concentration decreases.

This is the result of two processes: (1) changes in passive exchange of intraductal bicarbonate for intracellular chloride and (2) changes in the relative contribution of acinar cell secretion.

Acinar cells secrete fluid high in chloride in addition to digestive enzymes.

In contradistinction to anion concentrations, the concentrations of sodium and potassium in pancreatic duct secretions remain relatively constant despite the flow rate and are similar to their concentrations in plasma.

191
Q

Which pancreatic islet cell type produces a hormonal peptide
to stimulate glycogenolysis and gluconeogenesis?

A. Alpha cell
B. Beta cell
C. Delta cell
D. F cell
E. Pancreatic polypeptide (PP) cell
A

ANSWER: A

192
Q

Pancreatic delta cells secrete which inhibitory peptide?

A. Bombesin
B. Glucagon
C. Somatostatin
D. Insulin
E. Pancreatic polypeptide
A

ANSWER: C

COMMENTS: The endocrine pancreas is composed of various cells located in the islets of Langerhans, approximately 1 million of which are interspersed with the acinar and ductal elements throughout the gland.

The hormonal peptides produced by the islets affect a wide range of metabolic and physiologic actions.

The primary function of the endocrine pancreas is to regulate glucose homeostasis.

Beta cells, which are the most numerous, produce insulin. Insulin promotes glucose transport, stimulates protein synthesis, and inhibits glycogenolysis and lipolysis.

Alpha cells secrete glucagon, which counterbalances insulin by stimulating hepatic glycogenolysis, gluconeogenesis, ketogenesis, and lipolysis.

Glucagon also inhibits intestinal motility and gastric acid and pancreatic exocrine secretion.

Somatostatin, produced by delta cells, has a broad range of inhibitory effects on the gastrointestinal tract, including inhibition of secretion of other pancreatic peptides; inhibition of gastric, biliary, intestinal, and pancreatic exocrine secretions; and inhibition of gastrointestinal motility.

PP cells are the source of pancreatic polypeptide. Pancreatic polypeptide inhibits pancreatic exocrine secretion and biliary and gut motility.

Clinically, deficiency of pancreatic polypeptide has been linked to diabetes following resection of the pancreatic head or chronic pancreatitis.

Because postprandial secretion of pancreatic polypeptide is dependent on vagal innervation, it has been used to assess the completeness of vagotomy.

193
Q

Which of the following statements is true regarding blood flow to the pancreas?

A. Islet cells receive a greater proportion of pancreatic blood flow than do the exocrine elements.

B. CCK and secretin regulate secretion by altering the blood flow.

C. Fragile anastomotic networks predispose the gland to ischemia. D. The blood supply to the islet cells is independent of the acinar supply.

E. Pancreatic blood flow is highly sensitive to changes in systemic blood pressure.

A

ANSWER: A

COMMENTS: The microcirculation of the pancreas is complex and has important correlations with the endocrine and exocrine functions of the gland.

The rich anastomotic supply from various sources makes pancreatic ischemia unusual. The islets receive a disproportionately large amount of total pancreatic blood flow (10%–25%) relative to their mass (1%–2%).

Both the islets and exocrine tissue have an arteriolar blood supply.

The acinar tissue is also perfused by blood that drains from the islets, a mechanism referred to as the islet–acinar or insuloacinar portal system. This system is the structural basis for endocrine regulation of exocrine function.

Insulin receptors are present on acinar cells, and the density of receptors is higher on acini located near the islets.

Because the islets themselves often have a central-to-peripheral pattern of perfusion, insulin from the centrally located beta cells can influence the other peripheral islet cell types.

In addition, some islets are apparently perfused in a peripheral-to-central pattern.

CCK and secretin have relatively little effect on the blood flow and thus exert their stimulatory effects independently.

Pancreatic blood flow is maintained relatively con- stant despite changes in arterial pressure.

194
Q

What is the leading cause of death from acute pancreatitis?

A. Hemorrhage

B. Pseudocyst rupture

C. Secondary pancreatic infection

D. Biliary sepsis

E. Renal failure

A

ANSWER: C

COMMENTS: Formerly, death from acute pancreatitis often occurred early in the course of the disease as a result of the acute effects of hypovolemia and inadequate resuscitation.

In the current era, about 80% of deaths are attributed to secondary pancreatic infection, which develops in approximately 10% of patients with acute pancreatitis.

Fatal pancreatic sepsis typically progresses to multisystem organ failure, and deaths occur later in the course of the disease.

To have an impact on this disease, therapeutic efforts have therefore focused on the prevention and early diagnosis of pancreatic infection and on more effective methods of surgical therapy.

195
Q

A 58-year-old man with recurrent alcoholic pancreatitis presents to the ED with 3 days of worsening abdominal pain, nausea, and vomiting. Further questioning elicits symptoms including rigors and subjective fevers. His temperature is 102.5°F, WBC is 23,000 cells per MCL, heart rate is 105 beats/ min, and blood pressure is 118/64 mmHg. CT scan is obtained, which shows a hypoenhancing pancreas with necrosis and air bubbles, a small pseudocyst, and a second peripancreatic fluid collection. Which of the following complications of acute pancreatitis is associated with the highest mortality rate?

A. Peripancreatic abscess

B. Infected pancreatic pseudocyst

C. Infected pancreatic necrosis

D. Sterile pancreatic necrosis

E. Bile duct obstruction

A

ANSWER: C

COMMENTS: Retroperitoneal infection is a serious, often fatal, complication of acute pancreatitis.

The early literature pertaining to the local infectious sequelae of pancreatitis may be confusing because of nonselective use of the term “pancreatic abscess” to describe infectious complications, which vary in severity.

Pancreatic abscess best describes a localized collection of drainable pus in or around the pancreas.

Pancreatic abscess and infected pseudocyst can be treated effectively by external drainage, and the anticipated mortality rate for each is about 5%.

Pancreatic necrosis is a manifestation of severe pancreatitis.

When accompanied by infection, it has been associated with a mortality rate that may exceed 40%, which is higher than that for noninfected necrosis.

Infected pancreatic necrosis is treated by operative debridement and open or closed retroperitoneal drainage.

Patients with sterile necrosis may require operative intervention as well but are generally treated nonoperatively with intensive support as long as their condition permits.

196
Q

An alcoholic patient has acute pancreatitis with five of the Ranson criteria. He gradually improves over a 14-day hospitalization, but then a pulse of 120 beats/min, a temperature of 39°C, and abdominal distention develop. CT is performed, and the results are shown below (Fig. 26.2: Air in the pancreas). The next most appropriate therapy is which of the following measures?

A. Antibiotics

B. Percutaneous catheter drainage

C. Peritoneal lavage

D. Endoscopic cyst gastrostomy

E. Operative drainage

A

ANSWER: E

COMMENTS: Pancreatic infection complicating acute pancreatitis should be suspected in any patient who fails to improve following supportive medical therapy or improves but then demonstrates deterioration.

Pancreatic infection occasionally occurs early during the chronologic course of the disease, typically occurring later, as in the patient described.

CT is the best method for imaging the pancreas.

The results of CT in this patient demonstrate air in the pancreas, which is characteristic of a pancreatic infection.

The technique of dynamic pancreatography can identify ischemic areas of pancreas and is useful for evaluating patients who may have pancreatic necrosis.

Dynamic pancreatography is performed by serially imaging the pancreas after bolus injection of an intravenous contrast agent.

Percutaneous needle aspiration of fluid collections or necrotic areas found on CT can be performed to identify the presence of infection and guide therapeutic decisions about the need for drainage.

When pancreatic infection is present, operative drainage and debridement are indicated.

Interest has been focused on the selection of closed or open methods of operative drainage.

Minimal-access operative approaches are also used to drain and debride pancreatic necrosis in the hope of lowering morbidity in these ill patients.

Percutaneous catheters can drain thin fluid but are usually inadequate for the management of infected pancreatic necrosis.

Peritoneal lavage has been used early in the course of patients with severe acute pancreatitis.

Endoscopic cyst gastrostomy may be appropriate for some patients with pancreatic pseudocysts.

These latter two modalities have no role in the man- agement of infected pancreatic necrosis.

197
Q

A 45-year-old man is admitted with severe alcoholic pancreatitis. CT scan shows 20% pancreatic sterile necrosis. Which of the following statements best describes the current use of antibiotics for this patient?

A. Systemic antibiotics are not indicated unless his condition deteriorates.

B. Systemic antibiotics are indicated for coverage of gut-derived bacteria.

C. Systemic antibiotics are indicated for coverage of gut- derived bacteria and fungal organisms.

D. Nonabsorbable antibiotics are indicated for gut decon- tamination.

E. Systemic antibiotics are not indicated if enteric antibiotics can be tolerated.

A

ANSWER: A

COMMENTS: The risk for infected pancreatic necrosis is related to the clinical severity and duration of disease and to the extent of necrosis.

Strategies to decrease secondary pancreatic infection focus on patients at higher risk.

Unfortunately, controlled trials of systemic antibiotics for prophylaxis against secondary infection have yielded conflicting results.

These differences are probably because of numerous factors, including heterogeneity in the severity of disease, patient characteristics, and concomitant therapy among those studied, as well as differences in study methodologies.

Current practice favors systemic antibiotics for patients with severe disease and more extensive (>30%) necrosis based on studies demonstrating fewer septic complications and perhaps decreased mortality.

However, not all studies have shown benefit, and the risk for subsequent infection with multiresistant bacterial or fungal organisms may be increased, particularly if prophylactic antibiotic use is prolonged.

Because the gut is typically the source of the offending organisms, the use of nonabsorbable enteral antibiotics for selective gut decontamination has had some appeal.

The effect of this measure remains unclear, and it is not typically used.

Enteric feedings are beneficial to maintain the gut mucosal barrier to bacterial translocation.

However, the efficacy of enteric feeding alone for prevention of secondary pancreatic infection has not been demonstrated.

198
Q

A 54-year-old male presents to the ED with RUQ pain and epigastric pain and associated emesis. Lab results are significant for a total bilirubin of 1.9 g/dL, WBC of 11,500 cells per MCL, and lipase level of 300 U/L, and AST/ALT are mildly elevated. An US is performed showing a duct of 6 mm and numerous mobile stones in the gallbladder. Acute gallstone pancreatitis is diagnosed. Which of the following is considered standard treatment?

A. Urgent (within 24 h) cholecystectomy and common bile duct exploration

B. Urgent ERCP and subsequent laparoscopic cholecystectomy

C. Initial supportive therapy with cholecystectomy performed during the same admission

D. Initial supportive therapy with cholecystectomy per- formed in 6 to 8 weeks

E. Initial supportive therapy with cholecystectomy per- formed only if symptoms recur

A

ANSWER: C

COMMENTS: Gallstone pancreatitis is related to the passage of stones through the ampulla of Vater. Patients with smaller gallstones have an increased risk for the development of this manifestation.

Cholecystectomy is indicated because gallstone pancreatitis is a recurrent problem in 30%–50% of patients if surgery is not performed.

The traditional controversy has involved the timing of surgery. Proponents of immediate intervention have found a higher incidence of choledocholithiasis but have not demonstrated that this approach is safer than delayed surgery or that it is necessary for most patients.

Most surgeons advise initial nonoperative therapy until the patient’s signs and symptoms subside (most do within 2 to 3 days), followed by elective cholecystectomy with intraoperative imaging of the common bile duct by cholangiography or intraoperative ultrasonography during the same hospitalization.

The role of urgent endoscopic retrograde cholangiopancrea- tography and endoscopic sphincterotomy for the management of biliary pancreatitis has been controversial.

The vast majority (97%) of patients with gallstone pancreatitis have mild pancreatitis that improves rapidly.

ERCP finds common duct stones in only a small percentage of patients and is not indicated routinely.

Some trials comparing urgent ERCP and sphincterotomy with traditional treatment have suggested benefit in patients with severe pancreatitis, but this has not been consistently observed.

ERCP is indicated for patients with concomitant obstructive jaundice and biliary sepsis.

Less invasive methods of duct imaging, such as MRCP or EUS, might be useful in patients at an intermediate risk for choledocholithiasis but are not necessary for most with biliary pancreatitis.

For the small proportion of patients with severe biliary pancreatitis, early cholecystectomy should be avoided.

Treatment in this group is directed at resolution of the pancreatitis and its complications.

When the pancreatitis has subsided, delayed cholecystectomy is indicated.

199
Q

Which of the following is the preferred nutritional support for a patient with severe pancreatitis?

A. Nasogastric feeding

B. Feeding via percutaneous endoscopic gastrostomy

C. Nasojejunal feeding

D. Parenteral amino acids and glucose

E. Parenteral amino acids, glucose, and lipids

A

ANSWER: A

COMMENTS: Nutritional support is a critical component of the successful management of patients with severe pancreatitis.

Mortality is reduced by positive nitrogen balance.

Direct delivery of nutrients into the jejunum is the preferred route.

Enteral feeding helps maintain the intestinal mucosal barrier and prevent infection from early bacterial translocation.

Enteral feeding is associated with a lower risk for infection and shorter hospital stay than is parenteral nutrition.

Moreover, enteric feeding helps avoid catheter-related sepsis and other complications of central venous lines.

Recent studies have shown feeding into the stomach to be at least equal to nasojejunal feedings.

Oral feeding is cheaper and easier to manage than jejunal feedings. Nasojejunal tubes may require radiologically guided or endoscopic placement.

If nutritional goals cannot be met within a few days of initiation, parenteral nutritional support may also be necessary.

Intravenous lipids are not detrimental and prevent essential fatty acid deficiency.

200
Q

A 45-year-old nondiabetic patient with chronic alcoholic pancreatitis and intractable abdominal pain has extensive fibrosis of the intrapancreatic duct and distal common bile duct. The pancreatic duct measures 8 mm. Which of the following choices constitutes the best treatment?

A. Sphincteroplasty

B. Lateral pancreaticojejunostomy

C. Whipple procedure

D. Total pancreatectomy

E. Continued nonoperative therapy

A

ANSWER: C

COMMENTS: Pain is the primary indication for surgery in patients with chronic pancreatitis.

Selection of the best operation for a particular patient must include consideration of the anatomy of the gland, preexisting endocrine or exocrine dysfunction, compliance and the rehabilitative capacity of the patient, postoperative endocrine or exocrine deficiency, and the likelihood of postoperative pain relief.

Patients with small ducts disease are treated by resection if surgery is necessary.

Resection of the pancreatic head in properly selected patients has generally yielded better long-term results for pain relief than those yielded by tail resection.

The head of the pancreas is often enlarged and bulky in chronic pancreatitis and has been considered to be the “pacemaker” of the disease.

A number of operative techniques are available for resection of the pancreatic head including Whipple procedure.

This strategy is best employed in patients with calcified obstructive fibrosis of the bile duct, pancreatic duct, and/or duodenum.

Other patients with a dilated duct (>6 mm) and without focal findings restricted to the head of the pancreas are candidates for ductal drainage, with lateral pancreaticojejunostomy being the best choice for these procedures.

It is important to achieve adequate decompression of the enlarged pancreatic head and uncinate process during drainage procedures.

Variations such as the Frey or Beger procedure are intended to accomplish this.

The Frey procedure involves patients with increased disease in the head of the pancreas where portions of the head are cored out, and a lateral pancreaticojejunostomy is performed along the opened pancreatic duct.

The Beger procedure involves total excision of the head and neck of the pancreas, with a residual edge of pancreas preserving the duodenum and bile duct remains.

A loop of jejunum is then brought up, and distal pancreaticojejunostomy as well as a pancre- aticojejunostomy to the residual edge of the pancreas is performed.

Further, a jejunojejunostomy is performed for duodenal drainage. Sphincteroplasty does not play a role in the management of patients with established chronic pancreatitis.

Total or nearly total (95%) resections have higher long-term morbidity and mortality rates related to postoperative endocrine insufficiency.

Although endocrine and exocrine function tends to deteriorate over time in patients with chronic pancreatitis, some evidence suggests that pancreaticojejunostomy halts or delays this decline better than does nonoperative therapy.

201
Q

CT demonstrates a 5-cm peripancreatic fluid collection in a patient 3 weeks after an episode of acute pancreatitis. The patient is eating and does not have clinical signs of infection. What is the recommended treatment?

A. Expectant management without intervention

B. Nothing by mouth and total parenteral nutrition

C. Percutaneous catheter drainage of the fluid collection

D. Endoscopic drainage

E. Reimaging in 3 to 6 weeks and surgery for internal drainage if the collection persists

A

ANSWER: A

COMMENTS: Peripancreatic fluid collections can be found in about 20% of patients with acute pancreatitis. Many of them resolve spontaneously and should not be mistaken for pancreatic pseudocysts.

If the patient is stable, can eat, and does not have clinical evidence of infection or other complications, expectant management is indicated. The fluid collection can be monitored with ultrasonography or CT in 1 to 3 months.

If the patient has persistent pain and is unable to eat, nutrition by postpyloric enteral feeding, or parenteral nutrition if necessary, may be instituted for several weeks to allow resolution or maturation of the collection into a pseudocyst.

If the patient has a symptomatic or complicated fluid collection that requires early intervention, some method of external drainage must be used.

If the fluid is thin, endoscopic or percutaneous catheter drainage may suffice.

Operative drainage is preferred if there is substantial necrotic debris, as there often is, or if there is concern about infection. Operative drainage might be accomplished with minimal-access approaches.

202
Q

Which of the following is the most important determinant of
the need for drainage of a pancreatic pseudocyst?

A. Pseudocyst symptoms

B. Pseudocyst size

C. Pseudocyst duration

D. Associated chronic pancreatitis

E. Patient age

A

ANSWER: A

COMMENTS: Historically, pancreatic pseudocysts larger than 5 to 6 cm and present for longer than 6 weeks were thought to have a low rate of spontaneous resolution and a high rate of complications.

They were therefore treated by operative drainage.

Current understanding of the natural history of pseudocysts is that the rate of spontaneous resolution is higher and the rate of complications is lower than previously thought.

Pseudocyst size and duration are therefore no longer absolute criteria for intervention.

Rather, pseudocyst-related symptoms are the primary indication for treatment. Large pseudocysts are more likely to be symptomatic and less likely to resolve spontaneously compared with small pseudocysts.

In addition, pseudocysts in patients with chronic pancreatitis are unlikely to resolve but may not require intervention if they are stable, asymptomatic, and uncomplicated.

203
Q

A patient with chronic pancreatitis is unable to eat because of persistent postprandial pain. CT is performed (shown below). What is the recommended treatment?

A. Nothing by mouth and total parenteral nutrition for 4 to 6 weeks

B. Percutaneous catheter drainage

C. Endoscopic drainage

D. Operative internal drainage

E. Operative external drainage

A

ANSWER: D

COMMENTS: Pseudocysts that develop in patients with chronic pancreatitis can be considered mature when they are discovered unless there has also been a recent episode of acute pancreatitis.

The indications for treatment of a pancreatic pseudocyst are (1) persistent symptoms (pain, inability to eat, or biliary or gastrointestinal obstruction), (2) enlargement, or (3) the onset of a pseudocyst-related complication (infection, hemorrhage, or rupture).

Operative internal pseudocyst drainage into the stomach, jejunum, or duodenum is generally the preferred treatment, depending on the location of the pseudocyst.

For patients with chronic pancreatitis, it is critical to evaluate the pancreatic duct to determine whether a concomitant duct drainage procedure is necessary.

Pseudocyst drainage can be accomplished laparoscopically in some situations.

Pseudocysts in the tail of the gland are sometimes best treated by distal pancreatectomy.

Percutaneous or endoscopic drainage of established pseudocysts is still being debated.

These techniques can successfully treat pseudocysts in some circumstances but have definite limitations and potential complications.

204
Q

A 64-year-old man is evaluated for abdominal pain. CT shows segmental dilation of the main pancreatic duct to greater than 10 mm in the head of the gland with mural nodules. Which of the following is the next most appropriate recommendation?

A. EUS

B. ERCP

C. Serum cancer antigen 19-9

D. Total pancreatectomy

E. Abstinence from alcohol and CT repeated in 3 months

A

ANSWER: A
COMMENTS: An intraductal papillary mucinous neoplasm (IPMN) of the pancreas is a premalignant condition characterized by papillary projections of mucin-secreting epithelial cells, excessive mucin production, and cystic dilation of the pancreatic duct.

The patient may already have cancer when initially seen or may be at some other stage along the process of malignant transformation, which occurs relatively slowly.

IPMNs are divided into a main duct type and a branch duct type depending on the areas of the pancreatic ducts that are involved.

The goals of evaluation are to identify factors associated with a higher risk for malignancy and to determine the anatomic extent of the disease.

EUS is usually the next step in evaluation when IPMN is suspected. EUS can identify diffuse or segmental dilation of the pancreatic duct and the size of cystic lesions or mural nodules and can guide fine-needle aspiration to assess cytology and molecular tumor markers.

ERCP shows duct dilation without strictures, filling defects from mucus or nodules, and commonly a patulous papillary orifice with mucus.

ERCP also permits sampling of mucus and therapeutic clearance if mucus obstruction is a problem.

If available, direct pancreatoscopy and intraductal ultrasound can be adjuncts to ERCP for determining the extent of an IPMN.

205
Q

A 68-year-old male presents to his primary care physician (PCP) with abdominal pain in the epigastrium and associated nausea. No other significant findings are observed. The patient denies any alcohol or tobacco use. The PCP sends him to the ED for a CT scan, which shows a lesion in the pancreas. The patient subsequently undergoes an EUS/ ERCP, which shows mucin at the ampulla. MRI and biopsy results confirm a main duct–type IPMN. Which of the following features of IPMN of the pancreas is associated with the lowest risk for cancer?

A. Branch duct type with mural nodularity

B. Branch duct type smaller than 3 cm

C. Main duct type with diffuse dilation to greater than 10 mm

D. Main duct type with segmental dilation

E. Multifocal IPMN

A

ANSWER: B

COMMENTS: Progression of IPMN through the adenoma cancer sequence is considered a slow process that requires perhaps 10 to 20 years.

Main duct–type tumors have a greater risk for malignancy than the branch duct type. The Sendai Consensus Guidelines identified the following features as risk factors for cancer and as general indicators for resection: main pancreatic duct dilation to greater than 10 mm, cyst size larger than 3 cm, presence of mural nodules, and atypical cytology.

Additional risk factors include high-grade dysplasia, multifocal or synchronous tumors, and increasing cyst size during follow-up. In branch duct–type tumors, mural nodularity or atypical cytology may be more important determinants than size.

206
Q

A 58-year-old male is referred to you by his PCP for evaluation of pancreatic exocrine insufficiency. Upon evaluation of the patient, you suspect insufficiency due to malnutrition. You perform a standard secretin test. Which of the following test findings correlate with this diagnosis?

(Total Secretion Volume - Bicarbonate Secretion - Enzyme Secretion)

A. Normal - Normal - Decreased

B. Decreased - Decreased - Decreased

C. Increased - Normal - Normal

D. Decreased - Normal - Normal

E. Normal - Normal - Normal

A

ANSWER: A

COMMENTS: The secretin test of pancreatic exocrine insufficiency is considered the classic testing method.

The patient fasts overnight, and the next day, a double-lumen tube is placed into the duodenum through which basal secretion levels are collected for reference.

Next, an intravenous bolus of two units of secretin/kg is administered, followed by four 20-min collection periods.

Normal levels include 2 mL/kg of pancreatic fluid per hour, bicarbonate concentration of 80 mmol/L, bicarbonate output of 10 mmol/L/h, and amylase of 6 to 18 IU/kg.

Bicarbonate is the biggest discriminator between normal patients and dysfunction.

“A” represents malnutrition.

“B” represents end-stage pancreatitis or advanced pancreatic cancer.

“C” is hemochromatosis (associated Zollinger- Ellison syndrome or cirrhosis).

“D” is early pancreatic cancer.

Other tests that could be performed include 5,5-dimethyl-2,4-oxazolidinedione (DMO), Lundh, or the noninvasive triolein breath test.

207
Q

Which of the following statements is true regarding acute pancreatitis?

A. Twenty-five percent of patients with the diagnosis of idiopathic acute pancreatitis (IAP) will have a recurrence in their lifetime.

B. The diagnosis of IAP is made in up to 30% of cases of acute pancreatitis.

C. Pancreatic divisum is the most common congenital abnormality of the pancreas occurring in 15% of the general population and is a potential contributor to IAP.

D. Patients with CF have a 10%–15% incidence of pancreatitis during their lifetime, secondary to their disease.

E. None of the above.

A

ANSWER: B

COMMENTS: The etiology for acute pancreatitis is determined in 70%–90% of cases, and thus up to 30% of cases are deemed idiopathic.

Further extensive investigation is warranted as over 50% of IAP will recur, increasing a person’s risk of developing chronic pancreatitis.

Entities such as biliary sludge/microlithiasis, sphincter of Oddi dysfunction, pancreatic divisum, hereditary pancreatitis, CF, choledochocele, anomalous pancreaticobiliary junction, annular pancreas, and tumors can all be less obvious causes of acute pancreatitis in cases deemed “idiopathic.”

Pancreatic divisum is the most common congenital malformation of the pancreas in 5%–8% of the general population.

CF is an autosomal recessive disease where a mutation in a regulator gene encoding cyclic adenosine monophosphate regulates chloride ion channels.

This disease leads to pancreatitis by diminished pancreatic enzyme secretion in 2% of CF patients.

208
Q

Which of the following statements is true regarding chronic pancreatitis?

A. Patients with chronic pancreatitis tend to die from complications of smoking, diabetes, and chronic alcohol use.

B. There is no difference in the rate of pancreatic carcinoma between patients with chronic pancreatitis and the general population.

C. Pancreatic cancer in patients with chronic pancreatitis is the leading cause of mortality.

D. There is no difference in the
rate of extrapancreatic malignancies between patients with chronic pancreatitis and the general population.

E. None of the above.

A

ANSWER: A

COMMENTS: Patients with chronic pancreatitis have an overall higher rate of mortality compared with that of the general population.

The 20-year survival after diagnosis is approximately 69%–80% (three- to fourfold higher mortality than general population). Interestingly, the mortality is usually not due to the progression of their pancreatitis to carcinoma, but due to the comorbidities that often go along with patients who suffer from pancreatitis, including alcoholic cirrhosis, diabetes, and smoking-related cardiovascular disease.

Of note, patients with chronic pancreatitis have an elevated risk of extrapancreatic malignancy (10%–15%) affecting mostly the upper and lower airways as well as the gastrointestinal tract.

The rate of pancreatic cancer is noted to be 1.8% at 10 years postdiagnosis and 4% at 20 years, which is higher than that of the general population (approximately 1.5% lifetime risk).

209
Q

A 55-year-old woman with a history of ulcerative colitis presents to your office with a new onset of vague epigastric pain and jaundice. She denies any weight loss. To her knowledge, this has not happened before. Lab results are significant for a WBC of 12,000 cells per MCL and a total bilirubin of 2.0 mg/dL. You send her to the emergency room for a CT scan (see Fig. 26.3). Subsequent ERCP showed areas of irregular narrowing throughout the intrapancreatic common bile duct. No masses are identified on EUS. What is the initial treatment of choice for this patient?

A. Puestow procedure
B. Total pancreatectomy
C. Whipple procedure
D. Steroids
E. Frey procedure
A

ANSWER: D

COMMENTS: Autoimmune pancreatitis can be very difficult to diagnose and is the cause of acute pancreatitis in 4%–6% of cases. Patient age varies between 30 and 70 years, and it affects men more often than women.

The majority (70%–80%) present with painless jaundice.

Acute pain, typical of classic pancreatitis, is rare.

Additionally, one may see associated symptoms of pancreatic insufficiency, as this disease may have been ongoing without notice.

Systemic inflammation/ inflammatory reactions may be associated affecting kidney, lung, biliary tree, salivary glands, and gastrointestinal (GI) tract.

Ulcerative colitis is observed in 30% of cases of associated systemic disease.

On CT or US, a boggy, thick, “sausage-shaped” pancreas can be seen often with a rim of hypo- or hyperattenuation (early versus late).

Irregularities in the intrapancreatic portion of the common bile duct can also be seen.

Immunoglobulin G4 levels are thought to be associated with autoimmune pancreatitis, but using levels as a diagnostic marker is highly debated.

Antinuclear antibody levels may also be elevated, indicating an autoimmune process.

Steroids are considered as standard therapy for this etiology of pancreatitis (typically 30 to 40 mg of prednisone for 1 week).

210
Q

A 62-year-old male with an extensive history of recurrent alcoholic pancreatitis presents to the ED with progressively worsening shortness of breath. His last attack of pancreatitis was 9 months ago. At that time, a new fluid collection around the pancreas and extensive calcifications throughout the pancreas were noted. On examination, he is saturating 94% on room air. Auscultation of the lungs reveals absent breath sounds two-thirds of the way up his left lung fields. The right chest has breath sounds throughout. X-ray shows a large left pleural effusion. WBC count is 6,000 cells per MCL, and the patient is afebrile and hemodynamically stable. He has vague abdominal pain and had one episode of emesis in the ED. The ED resident orders a CT chest/abdomen displayed below (Fig. 26.4).
What is the most likely diagnosis?

A. Pneumonia
B. Sympathetic effusion
C. Pancreatic fistula
D. Reactive effusion
E. Diaphragmatic hernia
A

ANSWER: C

COMMENTS: Pancreatic–pleural fistula is a very rare entity and complication of chronic/recurrent pancreatitis.

This usually evolves from a disruption in the pancreatic duct (often posterior), followed by fluid tracking in the retroperitoneum and up along the aorta or esophagus into the pleural space.

Initial nonoperative therapy can include nil per os status, total parenteral nutrition, repeat thoracentesis, and octreotide.

Interventional therapy utilizing ERCP to stent the ductal defect (if present) may also be attempted.

Conservative treatment has an efficacy of 30%–60% and a recurrence rate of 15%. Operative therapy for failed conservative therapy, including duct ligation and resection or Roux-en- Y reconstruction, has an efficacy of 90% with an 18% recurrence.

Regardless of treatment modality, patients often require prolonged hospitalization.

Currently, treatment consists of several weeks of conservative management before progressing to surgical intervention.

211
Q

A 65-year-old male presents to the ED with two episodes of bloody emesis. His vitals are within normal limits other than mild tachycardia up to 110 beats/min. His hemoglobin is 9 g/dL (on prior lab testing, his baseline was 11 g/dL). His medical history is significant for hypertension, diabetes, 40-pack-year smoking history, alcohol abuse, and chronic pancreatitis. A nasogastric tube is placed, and some bright blood-tinged fluid is aspirated. No further episodes of emesis occur. A CT scan is obtained and shows a hypoechoic focus within the splenic vein and a calcified pancreas. He is sent for esophagogastroduodenoscopy, which is shown below (Fig. 26.5).
Which is the most definitive management?

A. Splenectomy
B. Gastric variceal banding
C. Propranolol
D. Porto-caval shunt 
E. Spleno-renal shunt
A

ANSWER: A

COMMENTS: Discussions over the role of splenectomy have evolved as the quality and frequency of CT scans have risen.

Splenic vein thrombosis is thought to occur in 7%–20% of patients with recurrent pancreatitis. This can lead to left-sided portal hypertension and subsequent development of gastric varices.

Clinically significant hemorrhage secondary to variceal bleeding occurs in only 4% of those with pancreatitis-induced splenic vein thrombosis.

Since not all patients who have splenic vein thrombosis will present with GI hemorrhage, prophylactic splenectomy has fallen out of favor.

Splenectomy is now reserved for those who present with hemorrhagic complications of splenic vein thrombosis.

212
Q

A 57-year-old male presents to the ED with complaints of epigastric abdominal fullness and occasional pain over the past 8 months with associated nausea. He confirms a long-standing history of acute on chronic pancreatitis secondary to alcohol abuse. A CT scan from a prior admission 10 months ago showed a large pancreatic pseudocyst. In the ED, he has one episode of blood-tinged emesis. He remains hemodynamically stable, with a hemoglobin of 10 g/ dL. A CT scan is obtained (see Fig. 26.6).

What is the appropriate next step in management?

A. Exploratory laparotomy

B. Interventional radiology (IR) angiography and embolization

C. Laparoscopic cystogastrostomy

D. Endoscopic cystogastrostomy

E. IR percutaneous drainage

A

ANSWER: B

COMMENTS: This patient has a diagnosis of splenic artery pseudoaneurysm with a likely fistulous connection to a pancreatic pseudocyst.

His nausea is likely due to gastric compression from this pseudocyst.

As the patient is hemodynamically stable, IR embolization of the aneurysm to prevent further bleeding is the most appropriate next step in management, followed by surgical cystogastrostomy once his bleeding risk is controlled.

Rupture is associated with 50% mortality.

Planning a cystogastrostomy without management of an associated pseudoaneurysm can lead to intraoperative hemorrhage, delayed hemorrhage, and an increased risk to the patient.

Pseudoaneurysms are rare complications (3%–10% incidence) caused by pancreatic enzymes eroding into the surrounding vasculature.

The splenic artery is most frequently involved, but reports of involvement of gastroduodenal, pancreaticoduodenal, gastric, and hepatic arteries exist.

213
Q

Of the following listed genetic mutations, which occurs least
frequently in the pathway of pancreatic carcinogenesis?

A. p16 oncogene mutation

B. p53 oncogene mutation

C. DPC4 tumor suppressor gene deletion

D. DNA mismatch repair gene deletion

E. BRCA1

A

ANSWER: D

COMMENTS: Several mutations are considered significant and contributory to the development and progression of pancreatic cancer.

The aforementioned mutations have all been shown to be involved in this process. Of the mutations mentioned, deletion of the DNA mismatch repair gene occurs least frequently at a rate of 4% in pancreatic cancer.

When mutations in DNA mismatch repair genes occur, errors in DNA coding are unable to be repaired.

Oncogene p16 is involved in phosphorylation of growth factors and regulatory proteins, and thus loss of this gene leads to unchecked cellular checkpoints. The p53 gene is well studied and involved in a variety of cancers.

Its gene product includes a DNA- binding protein that induces apoptosis, thus functioning as a cellular checkpoint.

This is inactivated in up to 75% of pancreatic neoplasms.

DPC4 is located on chromosome 18q, and this chromosome is missing in 90% of pancreatic cancers.

This gene product functions as a tumor suppressor and is specifically mutated in 50% of pancreatic cancers.

Of note, BRCA2, well known for its involvement in ovarian and breast cancers, has shown increased risk of pancreatic cancer in those with this mutation.

214
Q

Which of the following familial genetic syndromes is not
associated with pancreatic cancer?

A. Familial atypical multiple mole melanoma syndrome (FAMMM)

B. Familial adenomatous polyposis (FAP)

C. Hereditary nonpolyposis colorectal cancer (HNPCC)

D. Autosomal dominant polycystic kidney disease (ADPKD)

E. All of the above

A

ANSWER: D

COMMENTS: One in 71 people in the United States will develop pancreatic cancer in their lifetime; 5%–10% of these cancer occurrences can be associated with another well-known inherited disorder.

FAMMM is an autosomal dominant disorder associated with an increased risk of melanoma and dysplastic nevi.

It involves alteration in the gene encoding for p16 protein on chromosome 9p (involved in phosphorylation of growth factors and regulatory proteins, and thus loss of this gene leads to unchecked cellular checkpoints).

There is an associated 58% risk of developing pancreatic cancer by age 80 in those affected.

FAP is another autosomal dominant gene mutation involving the APC gene, with primary risk involving colon cancer.

Associated extracolonic malignancies involve duodenal, thyroid, hepatic, and pancreatic cancers.

HNPCC is again primarily associated with colon cancer, with lifetime risk ranging from 52%–82%.

The defect involves a mutation in DNA mismatch repair.

Extracolonic malignancies include endometrial, ovarian, gastric, and pancreatic cancers (1%–4%; associated medullary histology).

ADPKD has not been reliably shown to be associated with pancreatic cancer.

215
Q

A 52-year-old male and his family are referred to you for evaluation after the diagnosis of pancreatic adenocarcinoma. After a review of his imaging, the patient is observed to have a resectable disease and is a candidate for a Whipple procedure. They read on a website that the procedure can be performed in two ways. When discussing pylorus-preserving versus pylorus-resecting (standard) pancreaticoduodenectomy, which of the following statements is true?

A. On average, there is no difference in operative time between the two procedures.

B. Patients who undergo pylorus-preserving pancreaticoduodenectomy have a lower rate of pancreatic leak and fistula development.

C. Patients undergoing pylorus-resecting pancreaticoduodenectomy have less blood loss and less need for blood transfusions.

D. Patients with pylorus-preserving pancreaticoduodenectomy have a higher rate of delayed gastric emptying.

E. None of the above.

A

ANSWER: D

COMMENTS: Pancreatic cancer is the fourth leading cause of cancer deaths in men and fifth leading cause in women.

The classic Whipple procedure (involving antrectomy) is associated with operative mortality rates of 5% in expert hands at high-volume centers.

However, it is also associated with morbidity as high as 30%–40% including fistulas, early and late dumping, weight loss, and reflux.

An extensive Cochrane review and a meta-analysis of approximately eight randomized clinical trials both support that there is no difference in overall operative morbidity including fistulas, wound infection, postoperative bleeding, biliary leak, or pancreatic leak between the two techniques.

There is no difference in operative mortality or oncologic outcome based on procedure type as well. Pylorus-preserving Whipple procedure seems to have lower operative time, less intraoperative blood loss, and decreased need for transfusions compared with pylorus-resecting Whipple procedure, but it does have a higher rate of delayed gastric emptying.

216
Q

Which of the following is true regarding neoadjuvant
treatment for resectable pancreatic cancer?

A. It is associated with increased morbidity and mortality at 30 days postoperatively.

B. Radiation alone is considered as the optimal neoadjuvant therapy in this disease.

C. It is associated with a lower rate of pancreatic fistula and deep space infection postoperatively.

D. It is the standard of care.

E. None of the above.

A

ANSWER: C

COMMENTS: Often a debated topic, neoadjuvant versus surgery as the first strategy has been an issue of much interest in the pancreatic surgery community.

Some surgeons advocate surgery first in order to attack the disease process early, prevent spread, and work with virgin tissue planes.

Others advocate for a neoadjuvant approach for several reasons.

One is a possibly improved management of micrometastatic disease.

Another reason is to see which patients progress early versus those who would likely benefit from surgery without delaying time to chemotherapy for those who progress aggressively.

Clinical trials regarding the neoadjuvant approach boast improved overall survival; however, there is no true randomized trial comparing neoadjuvant to surgery first, as it is difficult to accrue patients when the two treatment arms are so drastically different.

A recent review of the American College of Surgeons-National Surgical Quality Improvement Program (ACS-NSQIP) data of over 1500 patients with pancreatic cancer does show that patients who underwent neoadjuvant therapy prior to surgi- cal resection had a lower rate of pancreatic fistula (7.3% vs. 15.4%) and deep space infection (3% vs. 10.3%) compared with the surgery- first patients. Also, overall 30-day postoperative morbidity and mortality are reported to be no different in the ACS-NSQIP data set.

There is no clear national consensus at this time as to which approach should be the standard of care.

217
Q

A 60-year-old female is referred to your junior partner for the diagnosis of IPMN. She asks you for advice regarding any other workup that she may have missed. You recommend a colonoscopy. A stage II colon cancer in the sigmoid colon is diagnosed. How should you proceed?

A. Colon resection followed by pancreas resection at a second surgery

B. Pancreas resection followed by colon resection at a second surgery

C. Neoadjuvant chemoradiation

D. Simultaneous resection of both the IPMN and colon
cancer

A

ANSWER: D

COMMENTS: Extrapancreatic malignancies have been observed more commonly in patients with IPMN than in those with ductal cancer of the pancreas or other cystic pancreatic neoplasms.

The reason for this is not known.

No particular genetic predisposition for IPMN has been identified. Extrapancreatic cancer has been reported in approximately one-third of patients with IPMN.

Gastric cancer and colon cancer have been the most frequent.

No specific guidelines exist, but based on these findings, screening upper gastrointestinal endoscopy and colonoscopy would be appropriate recommendations in these patients.

Should an extrapancreatic malignancy be identified, simultaneous resection is often indicated.

218
Q

Which of the following studies is the most effective in identifying and localizing pancreatic neuroendocrine neoplasms?

A. MRI

B. Positron emission tomography/CT

C. Octreotide scan

D. Endoscopy with EUS

E. Ultrasound

A

ANSWER: D

COMMENTS: EUS, in experienced hands, is associated with up to 93% sensitivity and 95% specificity when localizing a suspected neuroendocrine tumor of the pancreas, particularly in the head.

Another highly regarded imaging modality with high accuracy (>75%) is somatostatin receptor scintigraphy (SRS; octreotide scan).

The accuracy of CT scan alone ranges from 64% to 82%, and MRI is slightly better at above 74% sensitivity, similar to SRS.

Limits of EUS include the inability to evaluate the liver for disease.

Thus when using EUS to localize neuroendocrine tumors of the pancreas, SRS or another imaging modality would be needed to evaluate for metastatic disease.

219
Q

A 48-year-old female presents to the ED with her third
episode of blurred vision, palpitations, and weakness. Glucose level is 38 mg/dL. Her symptoms resolve with the administration of glucose. On her prior two presentations, her glucose levels were 35 and 42 mg/dL, respectively, and she was responsive to glucose administration. She is referred to you by the hospitalist for concerns of an insulinoma. Which is the first best diagnostic test?

A. Check proinsulin levels
B. Oral glucose tolerance test
C. Monitored fast
D. Check C-peptide levels
E. Hemoglobin A1c
A

ANSWER: C

COMMENTS: Insulinoma is the most common functional neoplasm of the endocrine pancreas and is evenly distributed throughout the pancreas; 10% of insulinomas are associated with MEN-1, 10% are malignant, and 90% are sporadic and solitary.

Patients present with symptoms of hypoglycemia during fasting, low levels of glucose less than 50 mg/dL, and relief of symptoms with exogenous glucose (referred to as “Whipple triad”).

Symptoms can include findings associated with catecholamine surge including palpitations, diaphoresis, and tachycardia.

Also, neurologic symptoms such as confusion, seizures, and irritability may be observed.

Differential diagnosis can include tumors, such as mesothelioma and sarcoma, adrenal insufficiency, and self-administration of insulin.

In order to make the diagnosis of insulinoma, first a 24-h monitored fast should be performed.

This includes glucose and insulin samples every 4 to 6 h in addition to samples at any time that symptoms appear during this period. Insulin-to-glucose ratios greater than 0.4 (normal are less than 0.3) after a prolonged fast are highly sensitive for the diagnosis.

Levels of C-peptide and proinsulin are elevated often in these patients as well but are not so in patients who self-administer insulin.

The absolute diagnosis is made with these six key features:

(1) displayed blood glucose levels ≤ 40 mg/dL,
(2) concomitant insulin levels ≥ 3 U/L,
(3) C-peptide levels ≥ 200 pmol/L,
(4) proinsulin levels ≥ 5 pmol/L,
(5) β-hydroxybutyrate levels ≤ 2.7 mmol/L, and
(6) absence of sulfonylurea (metabolites) in the plasma or urine.

220
Q

Drainage procedures for the pancreas?

A

Duval-Zollinger:
Caudal pancreaticojejunostomy

Partington-Rochelle:
Modification of the Puestow-Gillesby procedure of longitudinal pancreaticojejunostomy, that does not include a caudal pancreatectomy

Puestow-Gillesby:
Longitudinal roux-en-y pancreaticojejunostomy (invaginating anastomosis between pancreas and jejunum, after amputating the tail of the pancreas and opening the duct along its long axis)

221
Q

Resectional procedures for the pancreas?

A

Fry and Child:
95% distal pancreatectomy (preserves rim of the pancreas In the pancreaticoduodenal groove, along with Its associated blood vessels and distal CBD)

Kausch-Whipple:
Pancreaticoduodenectomy

Priestley:
Total pancreatectomy

222
Q

Hybrid surgical procedures for the pancreas?

A

Beger:
Duodenum-preserving pancreatic head resection (DPHR)

Frey and Smith:
Local resection of pancreatic head with longitudinal pancreaticojejunostomy (LR-LPJ)

Hamburg:
Modification of the LR-LPJ procedure (wider excavation of the pancreatic head is created In continuity with the dorsal dochotomy, followed by a single, side to side pancreaticojejunostomy)

Berne:
Modification of the Beger procedure (excavation of the central portion of the head without formal division of the pancreatic neck)

223
Q

Most Important causes of pancreatic pseudocyst?

A

Pancreatitis (75%)

Trauma (25%)

224
Q

The Whipple procedure involves?

A

1) Resection of pancreatic or duodenal primary
2) Gastrojejunostomy (or duodenojejunostomy)
3) Hepaticojejunostomy
4) Pancreaticojejunostomy
5) Cholecystectomy

225
Q

Palliative options for pancreatic CA?

A

1) Biliary obstruction
- Endoscopic biliary metal stent (preferred)
- Percutaneous biliary drainage with subsequent internalization
- Open biliary enteric bypass

2) Gastric outlet obstruction

Good performance status:

  • Gastrojejunostomy (Open/Lap) +/- tube jejunostomy
  • Consider enteral stent

Poor performance status:

  • Enteral stent
  • Percutaneous endoscopic gastrostomy (PEG) tube

3) Severe tumor-associated abdominal pain
- EUS-guided celiac plexus neurolysis (fluoroscopic or CT-guided if unavailable)
- Consider palliative radiation +/- chemo if not already given

4) Depression, pain, malnutrition
- Palliative medicine service evaluation
- Nutritional evaluation

5) Pancreatic Insufficiency
- Pancreatic enzyme replacement

6) Thromboembolic disease
- Low molecular weight heparin (preferred over warfarin)

226
Q

Whipple triad of insulinoma?

A

1) Symptomatic fasting hypoglycemia
2) Serum glucose <50mg/dL
3) Relief of symptoms with glucose administration

227
Q

Main treatment for insulinoma?

A

Simple enucleation

228
Q

Indications for Whipple procedure for insulinomas?

A

1) Close to main pancreatic duct

2) >2cm

229
Q

Where are VIPomas usually located?

A

Pancreatic tail

230
Q

Treatment for VIPomas?

A

Debulking (as palliative treatment)

231
Q

Common location for glucagonomas?

A

Body and tail

232
Q

Duval-Zollinger?

A

Caudal pancreaticojejunostomy

*Drainage

233
Q

Puestow-Gillesby?

A

Longitudinal roux-en-y pancreaticojejunostomy (invaginating anastomosis between pancreas and jejunum after amputating the tail of the pancreas, and opening the duct along its long axis)

*Drainage

234
Q

Partington-Rochelle?

A

Modification of the Puestow-Gillesby procedure of longitudinal pancreaticojejunostomy that does not include a caudal pancreatectomy

*Drainage

235
Q

Fry and Child?

A

95% distal pancreatectomy (preserves the rim of pancreas in the pancreaticoduodenal groove, along with its associated blood vessels and distal common bile duct)

*Resectional

236
Q

Kausch-Whipple?

A

Pancreaticoduodenectomy

*Resectional

237
Q

Priestley?

A

Total pancreatectomy

*Resectional

238
Q

Beger?

A

Duodenum-preserving pancreatic head resection (DPHR)

*Drainage+Resection

239
Q

Frey and Smith?

A

Local resection of the pancreatic head with longitudinal pancreaticojejunostomy (LR-LPJ)

*Drainage+Resection

240
Q

Hamburg?

A

Modification of the LR-LPJ procedure (wider excavation of the pancreatic head is created in continuity with the dorsal dichotomy followed by a single, side-to-side pancreaticojejunostomy)

*Drainage+Resection

241
Q

Berne?

A

Modification of the Beger procedure (excavation of the central portion of the head without formal division of the pancreatic neck)

242
Q

Most important causes of pancreatic pseudocysts?

A

Pancreatitis (75%)

Trauma (25%)

243
Q

Components/steps in a pancreaticoduodenectomy.

A

1) Exposure of infra pancreatic SMV
2) Extended Kocher maneuver
3) Portal dissection
4) Transect stomach (antrectomy)
5) Transect jejunum and dissect ligament of Treitz, rotating duodenum under mesenteric vessels
6) Transect pancreas and complete retroperitoneal dissection by removing specimen from SMV and SMA.

244
Q

Palliative options for biliary obstruction?

A

1) Endoscopic biliary metal stent (preferred)
2) Percutaneous biliary drainage
3) Open biliary enteric bypass

245
Q

Palliative options for gastric outlet obstruction?

A

Good performance status

1) Gastrojejunostomy +/- jejunostomy tube
2) Enteral stent

Poor performance status

1) Enteral stent
2) Percutaneous endoscopic gastrostomy (PEG) tube

246
Q

Palliative options for severe tumor-associated pain?

A

1) EUS-guided celiac plexus neurolysis (fluoroscopic or CT-guided if unavailable)
2) Palliative radiation +/- chemotherapy

247
Q

Palliative options for depression, pain, and malnutrition?

A

1) Palliative medicine service evaluation

2) Nutritional evaluation

248
Q

Palliative options for pancreatic insufficiency?

A

Pancreatic enzyme replacement

249
Q

Palliative options for thromboembolic disease?

A

Low molecular weight heparin (preferred over warfarin)

250
Q

Discuss Noninsulinoma Hyperinsulinemia Hypoglycemia Syndrome.

A
AKA NESIDEOBLASTOSIS:
1) Beta-cell hypertrophy
2) Islet-cell hyperplasia
3) Increased beta-cell mass
\+
1) Ectopic islet tissue
2) Multilobulated islets
3) Ductoinsular complexes

Nesideoblastosis with hyperinsulinism was previously considered a disease of neonates, where subtotal/total pancreatectomy was required to correct potentially fatal neonatal hyperinsulinism.

However, this has now been reported in patients 2-5 years after Roux-en-y gastric bypass for obesity.

This appears to result from an idiosyncratically-prolonged hyper secretion of the incretin hormones GIP and GLP-1 after the gastric bypass.

GLP-1 is a potent stimulant of the expression of the transcription factor PDX-1, which normally regulates beta-cell development and growth.

Treatment to prevent episodes of hypoglycemia is conversion of the gastric bypass to a form of bariatric procedure that restores normal intestinal flow of nutrients, such as the gastric sleeve, or the addition of a restriction element such as an adjustable gastric band.

251
Q

Discuss Non-functioning Pancreatic Endocrine Tumors.

A

AKA PET or pNET (pancreatic neuroendocrine tumors) are most often malignant, because they have the potential for uncontrolled growth and metastasis.

Dx:

1) Clinical
- Vague pain
- Weight loss

2) CT
- Incidental finding (asymptomatic)
- Enhances with arterial contrast
- Cystic component (central necrosis)

3) Somatostatin receptor scintigraphy (Octreoscan)
- Helpful in staging the disease

Tx:

1) Surgical resection with splenectomy
- Body and tail tumors

2) Observation
- small indolent nonfunctional pNETs (<2cm)

3) Adjuvant platinum-based chemotherapy
- for high-grade (poorly-differentiated) pNET

Follow-up:
- CT scan or ocreotide scan annually x 5 years after resection